Category Archives: Đa thức

PHÂN THỨC ĐẠI SỐ – P.2

 CÁC PHÉP TÍNH VỀ PHÂN THỨC

 

Muốn cộng các phân thức, ta quy đồng mẫu thức, cộng các tử thức với nhau, giữ nguyên mẫu thức chung, rồi rút gọn phân thức vừa tìm được.

Muốn trừ đi một phân thức, ta lấy phân thức bị trừ cộng với phân thức đối của phân thức trừ.

Muốn nhân các phân thức, ta nhân các tử thức với nhau, các mẫu thức với nhau, rồi rút gọn phân thức vừa tìm được. Muốn chia cho một phân thức khác 0 , ta lấy phân thức bị chia nhân với phân thức nghịch đảo của phân thức chia.

Ví dụ 1.

Cho $\mathrm{a}+\mathrm{b}+\mathrm{c}=0$ và $\mathrm{a}, \mathrm{b}, \mathrm{c}$ đều khạ́c 0 . Rút gọn biểu thức

$A=\frac{a b}{a^2+b^2-c^2}+\frac{b c}{b^2+c^2-a^2}+\frac{c a}{c^2+a^2-b^2} \text {. }$

Giải : Từ $\mathrm{a}+\mathrm{b}+\mathrm{c}=0$ suy ra $\mathrm{a}+\mathrm{b}=-\mathrm{c}$.

Bình phương hai vế, ta được $\mathrm{a}^2+\mathrm{b}^2+2 \mathrm{ab}=\mathrm{c}^2$ nên $\mathrm{a}^2+\mathrm{b}^2-\mathrm{c}^2=-2 \mathrm{ab}$.

Tương tự, $\mathrm{b}^2+\mathrm{c}^2-\mathrm{a}^2=-2 \mathrm{bc}$ và $\mathrm{c}^2+\mathrm{a}^2-\mathrm{b}^2=-2 \mathrm{ca}$.

Do đó $\mathrm{A}=\frac{\mathrm{ab}}{-2 \mathrm{ab}}+\frac{\mathrm{bc}}{-2 \mathrm{bc}}+\frac{\mathrm{ca}}{-2 \mathrm{ca}}=-\frac{1}{2}-\frac{1}{2}-\frac{1}{2}=-\frac{3}{2}$.

Ví dụ 2. Rút gọn biểu thức

$A=\frac{1}{1-x}+\frac{1}{1+x}+\frac{2}{1+x^2}+\frac{4}{1+x^4}+\frac{8}{1+x^8} .$

Giải : Do đặc điểm của bài toán, ta không quy đồng mẫu tất cả các phân thức mà cộng lần lượt từng phân thức.

$A =\frac{2}{1-x^2}+\frac{2}{1+x^2}+\frac{4}{1+x^4}+\frac{8}{1+x^8} $

$=\frac{4}{1-x^4}+\frac{4}{1+x^4}+\frac{8}{1+x^8}=\frac{8}{1-x^8}+\frac{8}{1+x^8}=\frac{16}{1-x^{16}}$

Ví dụ 3. Rút gọn biểu thức

$\mathrm{B}=\frac{3}{(1.2)^2}+\frac{5}{(2.3)^2}+\ldots+\frac{2 n+1}{[n(n+1)]^2}$

Giải : Đương nhiên không thể quy đồng mẫu tất cả các phân thức. Ta tìm cách tách mỗi phân thức thành hiệu của hai phân thức rồi dùng phương pháp khử liên tiếp. Ta có :

$\frac{2 \mathrm{k}+1}{\mathrm{k}^2(\mathrm{k}+1)^2}=\frac{(\mathrm{k}+1)^2-\mathrm{k}^2}{\mathrm{k}^2(\mathrm{k}+1)^2}=\frac{1}{\mathrm{k}^2}-\frac{1}{(\mathrm{k}+1)^2}$

Do đó : $\quad \mathrm{B}=\frac{1}{1^2}-\frac{1}{2^2}+\frac{1}{2^2}-\frac{1}{3^2}+\ldots+\frac{1}{\mathrm{n}^2}-\frac{1}{(\mathrm{n}+1)^2}=$

$=1-\frac{1}{(n+1)^2}=\frac{n(n+2)}{(n+1)^2}$

Ví dụ 4. Xác định các ‘số a, b, c sao cho

$\frac{1}{\left(x^2+1\right)(x-1)}=\frac{a x+b}{x^2+1}+\frac{c}{x-1} \text {. }\quad\quad(1)$

Giải : Thực hiện phép cộng ở vế phải của (1) :

$\frac{(a x+b)(x-1)+c\left(x^2+1\right)}{\left(x^2+1\right)(x-1)}=\frac{a x^2-a x+b x-b+c x^2+c}{\left(x^2+1\right)(x-1)}=$

$=\frac{(a+c) x^2+(b-a) x+(c-b)}{\left(x^2+1\right)(x-1)} \text {. }$

Đồng nhất phân thức trên với phân thức $\frac{1}{\left(x^2+1\right)(x-1)}$, ta được :

$\left\{\begin{array} { l }{ \mathrm { a } + \mathrm { c } = 0 } \\ { \mathrm { b } – \mathrm { a } = 0 } \\ { \mathrm { c } – \mathrm { b } = 1 }\end{array} \Rightarrow \left\{\begin{array}{l}\mathrm{c}+\mathrm{b}=0 \\ \mathrm{c}-\mathrm{b}=1\end{array} \Rightarrow \mathrm{c}=\frac{1}{2} ; \mathrm{b}=-\frac{1}{2} .\right.\right.$

Do đó $a=-\frac{1}{2}$. Như vậy : $\frac{1}{\left(x^2+1\right)(x-1)}=\frac{-\frac{1}{2} x-\frac{1}{2}}{x^2+1}+\frac{\frac{1}{2}}{x-1}$.

Ví dụ 5. Cho $\quad A=\frac{1}{(x+y)^3}\left(\frac{1}{x^4}-\frac{1}{y^4}\right)$, $B=\frac{2}{(x+y)^4}\left(\frac{1}{x^3}-\frac{1}{y^3}\right), \quad C=\frac{2}{(x+y)^5}\left(\frac{1}{x^2}-\frac{1}{y^2}\right)$.

Thực hiện phép tính $\mathrm{A}+\mathrm{B}+\mathrm{C}$.

Giải : Ta có

$A =\frac{y^4-x^4}{x^4 y^4(x+y)^3}=\frac{\left(y^2+x^2\right)\left(y^2-x^2\right)}{x^4 y^4(x+y)^3}=\frac{\left(y^2+x^2\right)(y-x)}{x^4 y^4(x+y)^2} $

$B+C =\frac{2}{(x+y)^4}\left(\frac{1}{x^3}-\frac{1}{y^3}+\frac{1}{x+y} \cdot \frac{y^2-x^2}{x^2 y^2}\right) $

$=\frac{2}{(x+y)^4}\left(\frac{1}{x^3}-\frac{1}{y^3}+\frac{y-x}{x^2 y^2}\right)=\frac{2}{(x+y)^4} \cdot \frac{y^3-x^3+x y(y-x)}{x^3 y^3}$

$=\frac{2}{(x+y)^4} \cdot \frac{(y-x)\left(y^2+2 x y+x^2\right)}{x^3 y^3}=\frac{2(y-x)}{(x+y)^2 x^3 y^3}$

Do đó $A+B+C=\frac{\left(y^2+x^2\right)(y-x)}{x^4 y^4(x+y)^2}+\frac{2(y-x)}{x^3 y^3(x+y)^2}=$

$=\frac{\left(y^2+x^2\right)(y-x)+2 x y(y-x)}{x^4 y^4(x+y)^2}=\frac{(y-x)\left(y^2+x^2+2 x y\right)}{x^4 y^4(x+y)^2}=\frac{y-x}{x^4 y^4}$

 

BÀI TẬP

19. Thực hiện phép tính :
a) $\frac{x+3}{x+1}-\frac{2 x-1}{x-1}-\frac{x-3}{x^2-1}$
b) $\frac{1}{x(x+y)}+\frac{1}{y(x+y)}+\frac{1}{x(x-y)}+\frac{1}{y(y-x)}$.
10. Thực hiện phép tính :
a) $A=\frac{1}{(a-b)(a-c)}+\frac{1}{(b-a)(b-c)}+\frac{1}{(c-a)(c-b)}$;
b) $B=\frac{1}{a(a-b)(a-c)}+\frac{1}{b(b-a)(b-c)}+\frac{1}{c(c-a)(c-b)}$;
c) $\mathrm{C}=\frac{\mathrm{bc}}{(\mathrm{a}-\mathrm{b})(\mathrm{a}-\mathrm{c})}+\frac{\mathrm{ac}}{(\mathrm{b}-\mathrm{a})(\mathrm{b}-\mathrm{c})}+\frac{\mathrm{ab}}{(\mathrm{c}-\mathrm{a})(\mathrm{c}-\mathrm{b})}$;
d) $D=\frac{a^2}{(a-b)(a-c)}+\frac{b^2}{(b-a)(b-c)}+\frac{c^2}{(c-a)(c-b)}$.
11. Cho $\mathrm{a}, \mathrm{b}, \mathrm{c}$ là các số nguyên khác nhau đôi một. Chứng minh rằng biểu thức sau có giá trị là một số nguyên :
$P=\frac{a^3}{(a-b)(a-c)}+\frac{b^3}{(b-a)(b-c)}+\frac{c^3}{(c-a)(c-b)}$

12. Cho $3 y-x=6$. Tính giá trị của biểu thức

$A=\frac{x}{y-2}+\frac{2 x-3 y}{x-6}$

13. Tìm $\mathrm{x}, \mathrm{y}, \mathrm{z}$, biết rằng $\frac{\mathrm{x}^2}{2}+\frac{\mathrm{y}^2}{3}+\frac{\mathrm{z}^2}{4}=\frac{\mathrm{x}^2+\mathrm{y}^2+\mathrm{z}^2}{5}$.

14. Tìm $\mathrm{x}, \mathrm{y}$, biết rằng $\mathrm{x}^2+\mathrm{y}^2+\frac{1}{\mathrm{x}^2}+\frac{1}{\mathrm{y}^2}=4$.

15. Cho biết :

$\frac{1}{\mathrm{a}}+\frac{1}{\mathrm{~b}}+\frac{1}{\mathrm{c}}=2$

$\frac{1}{\mathrm{a}^2}+\frac{1}{\mathrm{~b}^2}+\frac{1}{\mathrm{c}^2}=2 .$

Chứng minh rằng $\mathrm{a}+\mathrm{b}+\mathrm{c}=\mathrm{abc}$.

16. Cho

$\frac{\mathrm{x}}{\mathrm{a}}+\frac{\mathrm{y}}{\mathrm{b}}+\frac{\mathrm{z}}{\mathrm{c}}=0$

và $\quad \frac{\mathrm{a}}{\mathrm{x}}+\frac{\mathrm{b}}{\mathrm{y}}+\frac{\mathrm{c}}{\mathrm{z}}=2$.

Tính giá trị của biểu thức : $\frac{\mathrm{a}^2}{\mathrm{x}^2}+\frac{\mathrm{b}^2}{\mathrm{y}^2}+\frac{\mathrm{c}^2}{\mathrm{z}^2}$.

17. Cho $(a+b+c)^2=a^2+b^2+c^2$ và $a, b, c$ khác 0 . Chứng minh rằng

$\frac{1}{a^3}+\frac{1}{b^3}+\frac{1}{c^3}=\frac{3}{a b c}$

18. Cho

$\frac{\mathrm{a}}{\mathrm{b}}+\frac{\mathrm{b}}{\mathrm{c}}+\frac{\mathrm{c}}{\mathrm{a}}=\frac{\mathrm{b}}{\mathrm{a}}+\frac{\mathrm{a}}{\mathrm{c}}+\frac{\mathrm{c}}{\mathrm{b}}$

Chứng minh rằng trong ba số $\mathrm{a}, \mathrm{b}, \mathrm{c}$, tồn tại hai số bằng nhau.

19. Tìm các giá trị nguyên của $\mathrm{x}$ để phân thức sau có giá trị là số nguyên :

a) $\mathrm{A}=\frac{2 \mathrm{x}^3-6 \mathrm{x}^2+\mathrm{x}-8}{\mathrm{x}-3}$

b) $\mathrm{B}=\frac{\mathrm{x}^4-2 \mathrm{x}^3-3 \mathrm{x}^2+8 \mathrm{x}-1}{\mathrm{x}^2-2 \mathrm{x}+1}$

c) $C=\frac{x^4+3 x^3+2 x^2+6 x-2}{x^2+2}$

20. Rút gọn biểu thức sau với $\mathrm{x}=\frac{\mathrm{a}}{3 \mathrm{a}+2}$ :

$A=\frac{x+3 a}{2-x}+\frac{x-3 a}{2+x}-\frac{2 a}{4-x^2}+a$

21. Rút gọn biểu thức :

$A=\frac{2}{a-b}+\frac{2}{b-c}+\frac{2}{c-a}+\frac{(a-b)^2+(b-c)^2+(c-a)^2}{(a-b)(b-c)(c-a)} .$

  1. Cho biết $\frac{a+b-c}{a b}-\frac{b+c-a}{b c}-\frac{a+c-b}{a c}=0$. Chứng minh rằng trong ba phân thức ở vế trái, có ít nhất một phân thức bằng 0 .

23. Xác định các số a, b, c sao cho :

a) $\frac{1}{x\left(x^2+1\right)}=\frac{a}{x}+\frac{b x+c}{x^2+1}$

b) $\frac{1}{x^2-4}=\frac{a}{x-2}+\frac{b}{x+2}$

c) $\frac{1}{(x+1)^2(x+2)}=\frac{a}{x+1}+\frac{b}{(x+1)^2}+\frac{c}{x+2}$.

24. Rút gọn biểu thức

$\mathrm{B}=(\mathrm{ab}+\mathrm{bc}+\mathrm{ca})\left(\frac{1}{\mathrm{a}}+\frac{1}{\mathrm{~b}}+\frac{1}{\mathrm{c}}\right)-\mathrm{abc}\left(\frac{1}{\mathrm{a}^2}+\frac{1}{\mathrm{~b}^2}+\frac{1}{\mathrm{c}^2}\right)$

25. Cho $\mathrm{a}, \mathrm{b}, \mathrm{c}$ khác nhau đôi một và $\frac{1}{\mathrm{a}}+\frac{1}{\mathrm{~b}}+\frac{1}{\mathrm{c}}=0$. Rút gọn các biểu thức :

a) $M=\frac{1}{a^2+2 b c}+\frac{1}{b^2+2 a c}+\frac{1}{c^2+2 a b}$

b) $\mathrm{N}=\frac{\mathrm{bc}}{\mathrm{a}^2+2 \mathrm{bc}}+\frac{\mathrm{ca}}{\mathrm{b}^2+2 \mathrm{ac}}+\frac{\mathrm{ab}}{\mathrm{c}^2+2 \mathrm{ab}}$;

c) $\mathrm{P}=\frac{\mathrm{a}^2}{\mathrm{a}^2+2 \mathrm{bc}}+\frac{\mathrm{b}^2}{\mathrm{~b}^2+2 \mathrm{ac}}+\frac{\mathrm{c}^2}{\mathrm{c}^2+2 \mathrm{ab}}$.

26. Cho các số $a, b, c$ khác nhau đôi một và $\frac{a+b}{c}=\frac{b+c}{a}=\frac{c+a}{b}$. Tính giá trị của biểu thức

$\mathrm{M}=\left(1+\frac{\mathrm{a}}{\mathrm{b}}\right)\left(1+\frac{\mathrm{b}}{\mathrm{c}}\right)\left(1+\frac{\mathrm{c}}{\mathrm{a}}\right)$

27*. Cho $\mathrm{a}^3+\mathrm{b}^3+\mathrm{c}^3=3 \mathrm{abc}$ và $\mathrm{a}+\mathrm{b}+\mathrm{c} \neq 0$. Tính giá trị của biểu thức :

$N=\frac{a^2+b^2+c^2}{(a+b+c)^2}$

28. Rút gọn các biểu thức :

a) $A=\left(1-\frac{1}{2^2}\right)\left(1-\frac{1}{3^2}\right)\left(1-\frac{1}{4^2}\right) \ldots\left(1-\frac{1}{n^2}\right)$;

b) $\mathrm{B}=\frac{1^2}{2^2-1} \cdot \frac{3^2}{4^2-1} \cdot \frac{5^2}{6^2-1} \cdot \cdots \cdot \frac{(2 n+1)^2}{(2 n+2)^2-1} .$

29. Rút gọn các biểu thức :

a) $\frac{1}{1.2}+\frac{1}{2.3}+\frac{1}{3.4}+\ldots+\frac{1}{(n-1) n}$;

b) $\frac{1}{2.5}+\frac{1}{5.8}+\frac{1}{8.11}+\ldots+\frac{1}{(3 n+2)(3 n+5)}$;

c) $\frac{1}{1.2 .3}+\frac{1}{2.3 .4}+\frac{1}{3.4 .5}+\ldots+\frac{1}{(n-1) n(n+1)}$.

30. Chứng minh rằng với mọi số tự nhiên $\mathrm{n} \geq 1$ :

a) $\frac{1}{2^2}+\frac{1}{4^2}+\frac{1}{6^2}+\ldots+\frac{1}{(2 n)^2}<\frac{1}{2}$

b) $\frac{1}{3^2}+\frac{1}{5^2}+\frac{1}{7^2}+\ldots+\frac{1}{(2 n+1)^2}<\frac{1}{4}$.

31. Chứng minh rằng với mọi số tự nhiện $\mathrm{n} \geq 2$ :

$A=\frac{1}{2^2}+\frac{1}{3^2}+\frac{1}{4^2}+\ldots+\frac{1}{n^2}<\frac{2}{3} .$

32. Chứng minh rằng với mọi số tự nhiên $\mathrm{n} \geq 3$ :

$\mathrm{B}=\frac{1}{3^3}+\frac{1}{4^3}+\frac{1}{5^3}+\ldots+\frac{1}{\mathrm{n}^3}<\frac{1}{12} $

33. Chứng minh rằng với mọi số tự nhiên $\mathrm{n} \geq 1$ :

$A=\left(1+\frac{1}{1.3}\right)\left(1+\frac{1}{2.4}\right)\left(1+\frac{1}{3.5}\right) \cdots\left(1+\frac{1}{n(n+2)}\right)<2$

34. Chứng minh rằng với mọi số tự nhiên $\mathrm{n} \geq 2$ :

$\mathrm{B}=\left(1-\frac{2}{6}\right)\left(1-\frac{2}{12}\right)\left(1-\frac{2}{20}\right) \ldots\left(1-\frac{2}{\mathrm{n}(\mathrm{n}+1)}\right)>\frac{1}{3} \text {. }$

35. Rút gọn biểu thức

$A=\frac{3^2-1}{5^2-1} \cdot \frac{7^2-1}{9^2-1} \cdot \frac{11^2-1}{13^2-1} \cdot \ldots \frac{43^2-1}{45^2-1} .$

36*. Chứng minh rằng :

a) $\mathrm{A}=\frac{2^3+1}{2^3-1} \cdot \frac{3^3+1}{3^3-1} \cdot \frac{4^3+1}{4^3-1} \cdot \ldots \cdot \frac{9^3+1}{9^3-1}<\frac{3}{2}$.

b) $\mathrm{B}=\frac{2^3-1}{2^3+1} \cdot \frac{3^3-1}{3^3+1} \cdot \ldots \cdot \frac{\mathrm{n}^3-1}{\mathrm{n}^3+1}>\frac{2}{3}$.

37*. Rút gọn biểu thức

$P=\frac{\left(1^4+4\right)\left(5^4+4\right)\left(9^4+4\right) \ldots\left(21^4+4\right)}{\left(3^4+4\right)\left(7^4+4\right)\left(11^4+4\right) \ldots\left(23^4+4\right)} .$

38. Rút gọn biểu thức

$M=\frac{1}{a^2-5 a+6}+\frac{1}{a^2-7 a+12}+\frac{1}{a^2-9 a+20}+\frac{1}{a^2-11 a+30}$

39. Rút gọn biểu thức

9.$\left(\frac{\mathrm{n}-1}{1}+\frac{\mathrm{n}-2}{2}+\frac{\mathrm{n}-3}{3}+\ldots+\frac{2}{\mathrm{n}-2}+\frac{1}{\mathrm{n}-1}\right):\left(\frac{1}{2}+\frac{1}{3}+\frac{1}{4}+\ldots+\frac{1}{\mathrm{n}}\right) .$

40. Rút gọn biểu thức

$\frac{A}{B}=\frac{\frac{1}{1(2 n-1)}+\frac{1}{3(2 n-3)}+\frac{1}{5(2 n-5)}+\ldots+\frac{1}{(2 n-3) \cdot 3}+\frac{1}{(2 n-1) .1}}{1+\frac{1}{3}+\frac{1}{5}+\ldots+\frac{1}{2 n-1}} .$

41. Cho

$a b c=1$

và $\quad \mathrm{a}+\mathrm{b}+\mathrm{c}=\frac{1}{\mathrm{a}}+\frac{1}{\mathrm{~b}}+\frac{1}{\mathrm{c}}$.

Chứng minh rằng trong ba số a, b, c, tồn tại một số bằng 1 .

42. Chứng minh rằng nếu $\mathrm{x}+\mathrm{y}+\mathrm{z}=\mathrm{a}$ và $\frac{1}{\dot{\mathrm{x}}}+\frac{1}{\mathrm{y}}+\frac{1}{\mathrm{z}}=\frac{1}{\mathrm{a}}$ thì tồn tại một trong ba số $\mathrm{x}, \mathrm{y}, \mathrm{z}$ bằng $\mathrm{a}$.

43. Các biểu thức $\mathrm{x}+\mathrm{y}+\mathrm{z}$ và $\frac{1}{\mathrm{x}}+\frac{1}{\mathrm{y}}+\frac{1}{\mathrm{z}}$ có thể cùng có giá trị bằng 0 được hay không ?

44. Tính giá trị của biểu thức $\mathrm{M}=\frac{1}{\mathrm{x}+2}+\frac{1}{\mathrm{y}+2}+\frac{1}{\mathrm{z}+2}$, biết rằng $2 a=b y+c z, 2 b=a x+c z, 2 c=a x+b y$ và $a+b+c \neq 0$.

45. a) Cho abc $=2$. Rút gọn biểu thức

$M=\frac{a}{a b+a+2}+\frac{b}{b c+b+1}+\frac{2 c}{a c+2 c+2} .$

b) Cho abc $=1$. Rút gọn biểu thức

$\mathrm{N}=\frac{\mathrm{a}}{\mathrm{ab}+\mathrm{a}+1}+\frac{\mathrm{b}}{\mathrm{bc}+\mathrm{b}+1}+\frac{\mathrm{c}}{\mathrm{ac}+\mathrm{c}+1} .$

46. Cho $\frac{\mathrm{a}}{\mathrm{c}}=\frac{\mathrm{a}-\mathrm{b}}{\mathrm{b}-\mathrm{c}}, \mathrm{a} \neq 0, \mathrm{c} \neq 0, \mathrm{a}-\mathrm{b} \neq 0, \mathrm{~b}-\mathrm{c} \neq 0$. Chứng minh rằng

$\frac{1}{a}+\frac{1}{a-b}=\frac{1}{b-c}-\frac{1}{c}$

47. Cho $\mathrm{a}+\mathrm{b}+\mathrm{c}=0(\mathrm{a} \neq 0, \mathrm{~b} \neq 0, \mathrm{c} \neq 0)$. Rút gọn các biểu thức :

a) $\mathrm{A}=\frac{\mathrm{a}^2}{\mathrm{bc}}+\frac{\mathrm{b}^2}{\mathrm{ca}}+\frac{\mathrm{c}^2}{\mathrm{ab}}$

b) $\mathrm{B}=\frac{\mathrm{a}^2}{\mathrm{a}^2-\mathrm{b}^2-\mathrm{c}^2}+\frac{\mathrm{b}^2}{\mathrm{~b}^2-\mathrm{c}^2-\mathrm{a}^2}+\frac{\mathrm{c}^2}{\mathrm{c}^2-\mathrm{a}^2-\mathrm{b}^2}$.

48*. Tính giá trị của biểu thức sau, biết rằng $\mathrm{a}+\mathrm{b}+\mathrm{c}=0$ :

$A=\left(\frac{a-b}{c}+\frac{b-c}{a}+\frac{c-a}{b}\right)\left(\frac{c}{a-b}+\frac{a}{b-c}+\frac{b}{c-a}\right) \text {. }$

49. Chứng minh rằng nếu $\left(\mathrm{a}^2-\mathrm{bc}\right)(\mathrm{b}-\mathrm{abc})=\left(\mathrm{b}^2-\mathrm{ac}\right)(\mathrm{a}-\mathrm{abc})$ và các số $\mathrm{a}, \mathrm{b}, \mathrm{c}, \mathrm{a}-\mathrm{b}$ khác 0 thì $\frac{1}{\mathrm{a}}+\frac{1}{\mathrm{~b}}+\frac{1}{\mathrm{c}}=\mathrm{a}+\mathrm{b}+\mathrm{c}$.

50*. Cho $a+b+c=0, x+y+z=0, \frac{a}{x}+\frac{b}{y}+\frac{c}{z}=0$. Chứng minh rằng

$a x^2+b y^2+c z^2=0 .$

51. Cho $\frac{x y+1}{y}=\frac{y z+1}{z}=\frac{x z+1}{x}$. Chứng minh rằng $x=y=z$ hoặc $x^2 y^2 z^2=1$.

52. Cho $\frac{a}{b+c}+\frac{b}{c+a}+\frac{c}{a+b}=1$. Chứng minh rằng $\frac{a^2}{b+c}+\frac{b^2}{c+a}+\frac{c^2}{a+b}=0$.

53*. Cho $\frac{\mathrm{a}}{\mathrm{b}-\mathrm{c}}+\frac{\mathrm{b}}{\mathrm{c}-\mathrm{a}}+\frac{\mathrm{c}}{\mathrm{a}-\mathrm{b}}=0$. Chứng minh rằng

$\frac{a}{(b-c)^2}+\frac{b}{(c-a)^2}+\frac{c}{(a-b)^2}=0$

54. Cho $\mathrm{x}+\frac{1}{\mathrm{x}}=\mathrm{a}$. Tính các biểu thức sau theo $\mathrm{a}$ :

a) $x^2+\frac{1}{x^2}$

b) $x^3+\frac{1}{x^3}$

c) $x^4+\frac{1}{x^4}$

d) $x^5+\frac{1}{x^5}$

55. Cho $\left(x^2-\frac{1}{x^2}\right):\left(x^2+\frac{1}{x^2}\right)=a$. Tính biểu thức

$M=\left(x^4-\frac{1}{x^4}\right):\left(x^4+\frac{1}{x^4}\right) \text { theo } a$

  1. Cho $x^2-4 x+1=0$. Tính giá trị của biểu thức $A=\frac{x^4+x^2+1}{x^2}$.

57. Cho $\frac{x}{x^2-x+1}=a$. Tính $M=\frac{x^2}{x^4+x^2+1}$ theo $a$.

58. Cho $x=\frac{b^2+c^2-a^2}{2 b c}, y=\frac{a^2-(b-c)^2}{(b+c)^2-a^2}$.

Tính giá trị của biểu thức $\mathrm{x}+\mathrm{y}+\mathrm{xy}$.

59. Tìm hai số tự nhiên a và b sao cho :

a) $a-b=\frac{a}{b}$;

b) $a-b=\frac{a}{2 b}$

60. Cho hai số nguyên dương $\mathrm{a}$ và $\mathrm{b}$ trong đó $\mathrm{a}>\mathrm{b}$. Tìm số nguyên dương $\mathrm{c}$ khác b sao cho

$\frac{a^3+b^3}{a^3+c^3}=\frac{a+b}{a+c}$

61. Cho dãy số $a_1, a_2, a_3, \ldots$ sao cho :

$a_2=\frac{a_1-1}{a_1+1} ; a_3=\frac{a_2-1}{a_2+1} ; \ldots ; a_n=\frac{a_{n-1}-1}{a_{n-1}+1} .$

a) Chứng minh rằng $\mathrm{a}_1=\mathrm{a}_5$.

b) Xác định năm số đầu của dãy, biết rằng $\mathrm{a}_{101}=3$.

62. Tìm phân số $\frac{\mathrm{m}}{\mathrm{n}}$ khác 0 và số tự nhiên $\mathrm{k}$, biết rằng $\frac{\mathrm{m}}{\mathrm{n}}=\frac{\mathrm{m}+\mathrm{k}}{\mathrm{nk}}$.

63*. Cho hai số tự nhiên a và $\mathrm{b}(\mathrm{a}<\mathrm{b})$. Tìm tổng các phân số tối giản có mẫu bằng 7 , mỗi phân số lớn hơn a nhưng nhỏ hơn b.

64. a) Mức sản xuất của một xí nghiệp năm 2001 tăng a\% so với năm 2000, năm 2002 tăng b\% so với năm 2001. Mức sản xuất của xí nghiệp đó năm 2002 tăng so với năm 2000 là :

A) $(a+b) \%$;

B) $a b \%$

C) $\left(a+b+\frac{a+b}{100}\right) \%$

D) $\left(a+b+\frac{a b}{100}\right) \%$

$\mathrm{E})\left(\frac{\mathrm{a}+\mathrm{b}}{100}+\frac{\mathrm{ab}}{10000}\right) \%$

Hãy chọn câu trả lời đúng.

b) Một số a tăng m\%, sau đó lại giảm đi n\% ( $\mathrm{a}, \mathrm{m}, \mathrm{n}$ là các số dương) thì được số $b$. Tìm liên hệ giữa $m$ và $n$ để $b>a$.

65*. Chứng minh rằng các tổng sau không là số nguyên :

a) $\mathrm{A}=\frac{1}{2}+\frac{1}{3}+\frac{1}{4}+\ldots+\frac{1}{\mathrm{n}}(\mathrm{n} \in \mathrm{N}, \mathrm{n} \geq 2)$

b) $\mathrm{B}=\frac{1}{3}+\frac{1}{5}+\frac{1}{7}+\ldots+\frac{1}{2 \mathrm{n}+1}(\mathrm{n} \in \mathrm{N}, \mathrm{n} \geq 1)$.

 

CHUYÊN ĐỀ: TÍNH CHIA HẾT ĐỐI VỚI ĐA THỨC

Định lý Bezout và áp dụng

1. Đa thức chia có dạng $x-a$ (a là hằng)

Ví dụ 1. Chứng minh rằng số dư khi chia đa thức $\mathrm{f}(\mathrm{x})$ cho nhị thức $\mathrm{x}$ – a bằng giá trị của đa thức $\mathrm{f}(\mathrm{x})$ tại $\mathrm{x}=\mathrm{a}$.

Định lí Bê-du (Bézout, 1730 – 1783, nhà toán học Pháp).

Giải : Do đa thức chia $\mathrm{x}$ – a có bậc nhất nên số dư khi chia $\mathrm{f}(\mathrm{x})$ cho $\mathrm{x}-\mathrm{a}$ là hằng số $\mathrm{r}$.

Ta có $\quad \mathrm{f}(\mathrm{x})=(\mathrm{x}-\mathrm{a}) 、 \mathrm{Q}(\mathrm{x})+\mathrm{r}$.

Đẳng thức trên đúng với mọi $\mathrm{x}$ nên với $\mathrm{x}=\mathrm{a}$ ta có

$f(a)=0 . Q(a)+r \text { hay } f(a)=r \text {. }$

Chú ý : Từ định lí Bê-du ta suy ra :

Đa thức $\mathrm{f}(\mathrm{x})$ chia hết cho $\mathrm{x}-\mathrm{a}$ khi và chỉ khi $\mathrm{f}(\mathrm{a})=0$ (tức là khi và chỉ khi a là nghiệm của đa thức).

Ví dụ 2. Chứng minh rằng nếu đa thức $\mathrm{f}(\mathrm{x})$ có tổng các hệ số bằng 0 thì đa thức ấy chia hết cho $\mathrm{x}-1$ ‘.

Giải : Gọi : $f(x)=a_ox^n+a_1 x^n-1+\ldots+a_n-1x+a_n$.

Theo giả thiết, $\quad a_0+a_1+\ldots+a_{n-1}+a_n=0 $.

Theo định lí Bê-du, số dư khi chia $\mathrm{f}(\mathrm{x})$ cho $\mathrm{x}-1$ là

$r = f(1) = a_\circ + a_1 + \ldots + a_{n-1} + a_n $

Từ (1) và (2) suy ra $r=0$. Vậy $\mathrm{f}(\mathrm{x})$ chia hết cho $\mathrm{x}-1$.

Ví dụ 3. Chứng minh rằng nếu đa thức $\mathrm{f}(\mathrm{x})$ có tổng các hệ số của các hạng tử bậc chẵn bằng tổng các hệ số của các hạng tử bậc lẻ thì đa thức ấy chia hết cho $x+1$.

Giải : Gọi $f(x)=a_0 x^{2 n}+a_1 x^{2 n-1}+a_2 x^{2 n-2}+\ldots+a_{2 n-2} x^2+a_{2 n-1} x+a_{2 n}$, trong đó $\mathrm{a}_0$ có thể bằng 0 .

Theo giả thiết

$a_\circ + a_2 + \ldots + a_{2n} = a_2 + a_3 + \ldots + a_{2n-1}$ nên

$\left(a_0+a_2+\ldots+a_{2 n}\right)-\left(a_1+a_3+\ldots+a_{2 n-1}\right)=0 .$

Theo định lí Bê-du, số dư khi chia $\mathrm{f}(\mathrm{x})$ cho $\mathrm{x}+1$ bằng

$r =f(-1)=a_0-a_1+a_2-\ldots+a_{2 n-2}-a_{2 n-1}+a_{2 n} $

$=\left(a_o+a_2+\ldots+a_{2 n}\right)-\left(a_1+a_3+\ldots+a_{2 n-1}\right) $

Từ (1) và (2) suy ra $\mathrm{r}=0$. Vậy $\mathrm{f}(\mathrm{x})$ chia hết cho $\mathrm{x}+1$.

CHUYÊN ĐỀ: MỘT SỐ PHƯƠNG PHÁP PHÂN TÍCH ĐA THỨC THÀNH NHÂN TỬ

Trong chuyên đề này, ta chỉ phân tích đa thức thành nhân tử với các hệ số nguyên.

 

PHƯƠNG PHÁP TÁCH MỘT HẠNG TỬ THÀNH NHIỀU HẠNG TỬ

 

Ví dụ 1. Phân tích đa thức thành nhân tử :

$\quad\quad\quad\quad\quad\quad\quad\quad3 x^2-8 x+4$

Giải : Đa thức trên không chứa nhân tử chung, không có dạng một hằng đẳng thức đáng nhớ nào, cũng không thể nhóm các hạng tử. Ta biến đổi đa thức ấy thành đa thức có nhiều hạng tử hơn.

Cách 1. (Tách hạng tử thứ hai)

$3 x^2-8 x+4=3 x^2-6 x-2 x+4=3 x(x-2)-2(x-2)=(x-2)(3 x-2) \text {. }$

Cách 2. (Tách hạng tử thứ nhất)

$3 x^2-8 x+4=4 x^2-8 x+4-x^2=(2 x-2)^2-x^2 $

$=(2 x-2+x)(2 x-2-x)=(3 x-2)(x-2) .$

Nhận xét : Trong cách 1 , hạng tử $-8 \mathrm{x}$ được tách thành hai hạng tử $-6 \mathrm{x}$ và $-2 x$. Trong đa thức $3 x^2-6 x-2 x+4$, hệ số của các hạng tử là $3,-6,-2,4$. Các hệ số thứ hai và thứ tư đều gấp $-2$ lần hệ số liền trước, nhờ đó mà xuất hiện nhân tử chung $x-2$.

Một cách tổng quát, để phân tích tam thức bậc hai $\mathrm{ax}^2+\mathrm{bx}+\mathrm{c}$ thành nhân tử, ta tách hạng tử bx thành $b_1 x+b_2 x$ sao cho $\frac{b_1}{a}=\frac{c}{b_2}$, tức là $b_1 b_2=a c$.

Trong thực hành ta làm như sau :

Bước I : Tìm tích ac.

Bước 2 : Phân tích ac ra tích của hai thừa số nguyên bằng mọi cách.

Bước 3 : Chọn hai thừa số mà tổng bằng $\mathrm{b}$.

Trong ví dụ trên, đa thức $3 \mathrm{x}^2-8 \mathrm{x}+4$ có $\mathrm{a}=3, \mathrm{~b}=-8, \mathrm{c}=4$. Tích $\mathrm{ac}=3.4=12$. Phân tích 12 ra tích của hai thừa số, hai thừa số này cùng dấu (vì tích của chúng bằng 12 ), và cùng âm (để tổng của chúng bằng $-8)$ : $(-1)(-12)$, $(-2)(-6),(-3)(-4)$. Chọn hai thừa số mà tổng bằng $-8$, đó là $-2$ và $-6$.

Ví dụ 2. Phân tích đa thức thành nhân tử :

$\quad\quad\quad\quad\quad\quad\quad\quad4 x^2-4 x-3$

Giải :

Cách 1. (Tách hạng tử thứ hai)

$4 \mathrm{x}^2-4 \mathrm{x}-3=4 \mathrm{x}^2+2 \mathrm{x}-6 \mathrm{x}-3=2 \mathrm{x}(2 \mathrm{x}+1)-3(2 \mathrm{x}+1)=(2 \mathrm{x}+1)(2 \mathrm{x}-3)$

Chú ý rằng hệ số $-4$ được tách thành 2 và $-6$ có tích bằng $-12$, bằng tích của $4(-3)$.

Cách 2. (Tách hạng tử thứ ba)

$4 x^2-4 x-3=4 x^2-4 x+1-4=(2 x-1)^2-2^2=(2 x+1)(2 x-3)$

Nhận xét : Qua hai ví dụ trên, ta thấy việc tách một hạng tử thành nhiều hạng tử khác thường nhằm mục đích

  • Làm xuất hiện các hệ số tỉ lệ, nhờ đó mà xuất hiện nhân tử chung (cách 1 ) ;

  • Làm xuất hiện hiệu của hai bình phương (cách 2).

Với các đa thức có bậc từ bậc ba trở lên, để dễ dàng làm xuất hiện các hệ số tỉ lệ, người ta thường dùng cách tìm nghiệm của đa thức.

Ta nhắc lại khái niệm nghiệm của đa thức : số a được gọi là nghiệm của đa thức $\mathrm{f}(\mathrm{x})$ nếu $\mathrm{f}(\mathrm{a})=0$. Như vậy, nếu đa thức $\mathrm{f}(\mathrm{x})$ có nghiệm $\mathrm{x}=\mathrm{a}$ thì nó chứa nhân tử $x-a$.

Ta chứng minh được rằng nghiệm nguyên của đa thức, nếu có, phải là ước của hệ số tự do.

Thật vậy, giả sử đa thức $a_0 x^n+a_1 x^{n-1}+\ldots+a_{n-1} x+a_n$ với các hệ số $\mathrm{a}_{\mathrm{O}}$, $a_1, \ldots, a_n$ nguyên, có nghiệm $x=a(a \in \mathbf{Z})$ . Thế thì

$a_0 x^n+a_1 x^{n-1}+\ldots+a_{n-1} x+a_n=(x-a)\left(b_0 x^{n-1}+b_1 x^{n-2}+\ldots+b_{n-1}\right)$

trong đó $b_0, b_1, \ldots, b_{n-1}$ nguyên. Hạng tử có bậc thấp nhất của tích ở vế phải bằng $-a b_{n-1}$ . Do đó $-a b_{n-1}=a_n$, tức a là ước của $a_n$

Ví dụ 3. Phân tích đa thức thành nhân tử :

$\quad\quad\quad\quad\quad\quad\quad\quad f(x)=x^3-x^2-4$

Giải : Lần lượt kiểm tra với $\mathrm{x}=\pm 1, \pm 2, \pm 4$, ta thấy $\mathrm{f}(2)=2^3-2^2-4=0$. Đa thức có nghiệm $\mathrm{x}=2$, do đó chứa nhân tử $\mathrm{x}-2$.

Ta tách các hạng tử như sau :

Cách 1. $\quad \mathrm{x}^3-\mathrm{x}^2-4=\mathrm{x}^3-2 \mathrm{x}^2+\mathrm{x}^2-2 \mathrm{x}+2 \mathrm{x}-4$

$=x^2(x-2)+x(x-2)+2(x-2)=(x-2)\left(x^2+x+2\right)$

Cách 2. $\quad \mathrm{x}^3-\mathrm{x}^2-4=\mathrm{x}^3-8-\mathrm{x}^2+4$

$=(x-2)\left(x^2+2 x+4\right)-(x+2)(x-2) $

$=(x-2)\left(x^2+2 x+4-x-2\right)=(x-2)\left(x^2+x+2\right)$

Chú ý : Khi xét nghiệm nguyên của đa thức, nên nhớ hai định lí sau :

a) Nếu đa thức $\mathrm{f}(\mathrm{x})$ có tổng các hệ số bằng 0 thì 1 là nghiệm của đa thức, do đó đa thức chứa nhân tử $\mathrm{x}-1$.

Chẳng hạn, đa thức $\mathrm{x}^3-5 \mathrm{x}^2+8 \mathrm{x}-4$ có $1-5+8-4=0$ nên 1 là nghiệm của đa thức, đa thức chứa nhân tử $\mathrm{x}-1$.

b) Nếu đa thức $\mathrm{f}(\mathrm{x})$ có tổng các hệ số của hạng tử bậc chẵn bằng tổng các hệ số của hạng tử bậc lẻ thì $-1$ là nghiệm của đa thức, đa thức chứa nhân tử $\mathrm{x}+1$.

Chẳng hạn, đa thức $x^3-5 x^2+3 x+9$ có $9-5=3+1$ nên $-1$ là nghiệm của đa thức, đa thức chứa nhân tử $\mathrm{x}+1$.

Chú ý : Để nhanh chóng loại trừ các ước của hệ số tự do không là nghiệm của đa thức, có thể dùng nhận xét sau :

Nếu a là nghiệm nguyên của đa thức $\mathrm{f}(\mathrm{x})$ và $\mathrm{f}(1), \mathrm{f}(-1)$ khác 0 thì $\frac{\mathrm{f}(1)}{\mathrm{a}-1}$ và $\frac{\mathrm{f}(-1)}{\mathrm{a}+1}$ đều là số nguyên.

Chứng̉ minh. Số a là nghiệm của $\mathrm{f}(\mathrm{x})$ nên

$f(x)=(x-a) \cdot Q(x) \quad\quad(1)$

Thay $x=1$ vào (1), ta có $f(1)=(1-a) \cdot Q(1)$.

Do $\mathrm{f}(1) \neq 0$ nên $\mathrm{a} \neq 1$, vì thế $\mathrm{Q}(1)=\frac{\mathrm{f}(1)}{1-\mathrm{a}}$, tức là $\frac{\mathrm{f}(1)}{\mathrm{a}-1}$ là số nguyên.

Thay $\mathrm{x}=-1$ vào (1). Chứng minh tương tự, ta cũng có $\frac{\mathrm{f}(-1)}{a+l}$ là số nguyên. Lấy một ví dụ : $\quad \mathrm{f}(\mathrm{x})=4 \mathrm{x}^3-13 \mathrm{x}^2+9 \mathrm{x}-18$.

Các ước của 18 là $\pm 1, \pm 2, \pm 3, \pm 6, \pm 9, \pm 18$.

$f(1)=4-13+9-18=-18, f(-1)=-4-13-9-18=-44 .$

Hiển nhiên $\pm 1$ không là nghiệm của $f(x)$. Ta thấy $\frac{-18}{-3-1}, \frac{-18}{\pm 6-1}, \frac{-18}{\pm 9-1}$, $\frac{-18}{\pm 18-1}$ không nguyên nên $-3, \pm 6, \pm 9, \pm 18$ không là nghiệm của $f(x)$.

Ta thấy $\frac{-44}{2+1}$ không nguyên nên 2 không là nghiệm của $f(x)$. Chỉ còn $-2$ và 3 .

Kiểm tra ta thấy 3 là nghiệm của $\mathrm{f}(\mathrm{x})$. Do đó, ta tách các hạng tử như sau :

$ 4 x^3-13 x^2+9 x-18=4 x^3-12 x^2-x^2+3 x+6 x-18 $

$= 4 x^2(x-3)-x(x-3)+6(x-3)=(x-3)\left(4 x^2-x+6\right)$

Ví dụ 3. Phân tích đa thức thành nhân tử :

$\quad\quad\quad\quad\quad\quad\quad\quad3 x^3-7 x^2+17 x-5$

Giải : Các số $\pm 1, \pm 5$ không là nghiệm của đa thức. Như vậy, đa thức không có nghiệm nguyên. Tuy vậy, đa thức có thể có nghiệm hữu tỉ khác. Ta chứng minh được rằng trong đa thức có các hệ số nguyên, nghiệm hữu tỉ (nếu có) phải có dạng $\frac{\mathrm{p}}{\mathrm{q}}$ trong đó $\mathrm{p}$ là ước của hệ số tự do, $\mathrm{q}$ là ước dương của hệ số cao nhất (*).

Xét các số $\pm \frac{1}{3}, \pm \frac{5}{3}$, ta thấy $\frac{1}{3}$ là nghiệm của đa thức, do đó đa thức chứa thừa số $3 x-1$. Ta tách các hạng tử như sau :

$3 x^3-7 x^2+17 x-5=3 x^3-x^2-6 x^2+2 x+15 x-5 $

$= x^2(3 x-1)-2 x(3 x-1)+5(3 x-1)=(3 x-1)\left(x^2-2 x+5\right)$

(*) $-$ Thật vậy, giả sử đa thức $a_0 x^n+a_1 x^{n-1}+\ldots+a_{n-1} x+a_n$ với các hệ số $a_0, a_1, \ldots, a_n$ nguyên, có nghiệm hữu tỉ $x=\frac{p}{q}$, trong đó $p, q \in \mathbf{Z}, \mathrm{q}>0,(\mathrm{p}, \mathrm{q})=1$. Thế thì

$a_0 x^n+a_1 x^{n-1}+\ldots+a_{n-1} x+a_n=(q x-p)\left(b_0 x^{n-1}+b_1 x^{n-2}+\ldots+b_{n-1}\right)$

Ta có $-pb_n-1=a_n$, $qb_o=a_o$ nên p là ước của $a_n$; còn q là ước dương của $a_o$.

 

PHƯƠNG PHÁP THÊM VÀ BỚT CÙNG MỘT HẠNG TỬ

 

1. Thêm và bớt cùng một hạng tử làm xuất hiện hiệu của hai bình phương

Ví dụ 4. Phân tích đa thức thành nhân tử :

$\quad\quad\quad\quad\quad\quad\quad\quad4 x^4+81$

Giải : Thêm và bớt $36 \mathrm{x}^2$ :

$4 \mathrm{x}^4+81=4 \mathrm{x}^4+36 \mathrm{x}^2+81-36 \mathrm{x}^2$

$=\left(2 x^2+9\right)^2-(6 x)^2=\left(2 x^2+9+6 x\right)\left(2 x^2+9-6 x\right) .$

Ví dụ 5. Phân tích đa thức thành nhân tử :

$\quad\quad\quad\quad\quad\quad\quad\quad 64 x^4+y^4 \text {. }$

Giải : Thêm và bớt $16 \mathrm{x}^2 \mathrm{y}^2$ :

$64 x^4+y^4 =64 x^4+16 x^2 y^2+y^4-16 x^2 y^2=\left(8 x^2+y^2\right)^2-(4 x y)^2 $

$=\left(8 x^2+y^2+4 x y\right)\left(8 x^2+y^2-4 x y\right)$

2. Thêm và bớt cùng một hạng tử làm xuất hiện nhân tử chung

Ví dụ 6. Phân tích đa thức thành nhân tử :

$\quad\quad\quad\quad\quad\quad\quad\quad x^5+x-1$

Giải :

Cách 1:

$x^5+x-1 =x^5-x^4+x^3+x^4-x^3+x^2-x^2+x-1 $

$=x^3\left(x^2-x+1\right)+x^2\left(x^2-x+1\right)-\left(x^2-x+1\right) $

$=\left(x^2-x+1\right)\left(x^3+x^2-1\right)$

Cách 2. Thêm và bớt $\mathrm{x}^2$ :

$x^5+x-1=x^5+x^2-x^2+x-1=x^2\left(x^3+1\right)-\left(x^2-x+1\right) $

$=\left(x^2-x+1\right)\left[x^2(x+1)-1\right]=\left(x^2-x+1\right)\left(x^3+x^2-1\right)$

Ví dụ 7. Phân tích đa thức thành nhân tử :

$\quad\quad\quad\quad\quad\quad\quad\quad x^7+x^2+1$

Giải : Thêm và bớt x :

$x^7+x^2+1 =x^7-x+x^2+x+1 $

$=x\left(x^3+1\right)\left(x^3-1\right)+\left(x^2+x+1\right) $

$=x\left(x^3+1\right)(x-1)\left(x^2+x+1\right)+\left(x^2+x+1\right) $

$=\left(x^2+x+1\right)\left(x^5-x^4+x^2-x+1\right)$

Chú ý : Các đa thức dạng $\mathrm{x}^{3 \mathrm{~m}+1}+\mathrm{x}^{3 \mathrm{n}+2}+1 \mathrm{nhu} \mathrm{x}^7+\mathrm{x}^2+1, \mathrm{x}^7+\mathrm{x}^5+1$, $x+x^5+1, x+x^8+1, \ldots$ đều chứa nhân tử $x^2+x+1$

PHƯƠNG PHÁP ĐỔI BIẾN

Ví dụ 8. Phân tích đa thức thành nhân tử :

$\quad\quad\quad\quad\quad\quad\quad\quad x(x+4)(x+6)(x+10)+128$

Giải :

$x(x+4)(x+6)(x+10)+128=\left(x^2+10 x\right)\left(x^2+10 x+24\right)+128$

Đặt $x^2+10 x+12=y$, đa thức đã cho có dạng :

$(y-12)(y+12)+128=y^2-16=(y+4)(y-4) $

$=\left(x^2+10 x+16\right)\left(x^2+10 x+8\right)=(x+2)(x+8)\left(x^2+10 x+8\right) .$

Nhận xét : Trong ví dụ trên, nhờ phương pháp đổi biến, ta đã đưa đa thức bậc bốn đối với $x$ thành đa thức bậc hai đối với y.

Ví dụ 9. Phân tích đa thức thành nhân tử :

$\quad\quad\quad\quad\quad\quad\quad\quad A=x^4+6 x^3+7 x^2-6 x+1$

Giải : Giả sử $\mathrm{x} \neq 0$. Ta viết đa thức dưới dạng :

$A=x^2\left(x^2+6 x+7-\frac{6}{x}+\frac{1}{x^2}\right)=x^2\left[\left(x^2+\frac{1}{x^2}\right)+6\left(x-\frac{1}{x}\right)+7\right] \text {. }$

Đặt $x-\frac{1}{x}=y$ thì $x^2+\frac{1}{x^2}=y^2+2$. Do đó

$A =x^2\left(y^2+2+6 y+7\right)=x^2(y+3)^2=(x y+3 x)^2 $

$=\left[x\left(x-\frac{1}{\dot{x}}\right)+3 x\right]^2=\left(x^2+3 x-1\right)^2$

Dạng phân tích này cũng đúng với $x=0$.

Chú ý : Có thể trình bày lời giải của ví dụ trên như sau :

$A =x^4+6 x^3-2 x^2+9 x^2-6 x+1 $

$=x^4+2 x^2(3 x-1)+(3 x-1)^2=\left(x^2+3 x-1\right)^2$

 

PHƯƠNG PHÁP HỆ SỐ BẤT ĐỊNH

Ví dụ 10. Phân tích đa thức thành nhân tử :

$\quad\quad\quad\quad\quad\quad\quad\quad x^4-6 x^3+12 x^2-14 x+3$

Giải : Các số $\pm 1, \pm 3$ không là nghiệm của đa thức, đa thức không có nghiệm nguyên, cũng không có nghiệm hữu tỉ. Như vậy nếu đa thức trè̀n phân tích được thành nhân tử thì phải có dạng $\left(x^2+a x+b\right)\left(x^2+c x+d\right)$. Phép nhân. này cho kết quả $\mathrm{x}^4+(\mathrm{a}+\mathrm{c}) \mathrm{x}^3+(\mathrm{ac}+\mathrm{b}+\mathrm{d}) \mathrm{x}^2+(\mathrm{ad}+\mathrm{bc}) \mathrm{x}+\mathrm{bd}$. Đồng nhất đa thức này với đa thức đã cho, ta được hệ điều kiện :

$\quad\quad\quad\quad\quad\quad\quad\quad\left\{\begin{array}{l}a+c=-6 \\ a c+b+d=12 \\ a d+b c=-14 \\ b d=3 .\end{array}\right.$

Xét bd $=3$ với $\mathrm{b}, \mathrm{d} \in \mathbf{Z}, \mathrm{b} \in{\pm 1, \pm 3}$. Với $\mathrm{b}=3$ thì $\mathrm{d}=1$, hệ điều kiện trên trở thành :

$\quad\quad\quad\quad\quad\quad\quad\quad\left\{\begin{array}{l}a+c=-6 \\ a c=8 \\ a+3 c=-14\end{array}\right.$

Suy ra $2 \mathrm{c}=-14-(-6)=-8$. Do đó $\mathrm{c}=-4$, $\mathrm{a}=-2$.

Vậy đa thức đã cho phân tich thành $\left(x^2-2 x+3\right)\left(x^2-4 x+1\right)$.

Chú ý : Ta trình bày lời giải của ví dụ trên như sau :

$x^4-6 x^3+12 x^2-14 x+3=$

$= x^4-4 x^3+x^2-2 x^3+8 x^2-2 x+3 x^2-12 x+3 $

$= x^2\left(x^2-4 x+1\right)-2 x\left(x^2-4 x+1\right)+3\left(x^2-4 x+1\right) $

$=\left(x^2-4 x+1\right)\left(x^2-2 x+3\right)$

PHƯƠNG PHÁP XÉT GIÁ TRỊ RIÊNG

Trong phương pháp này, trước hết ta xác định dạng các nhân tử chứa biến của đa thức, rồi gán cho các biến các giá trị cụ thể để xác định nhân tử còn lại.

Ví dụ 11. Phân tích đa thức thành nhân tử :

$\quad\quad\quad\quad\quad\quad\quad\quad P=x^2(y-z)+y^2(z-x)+z^2(x-y)$

Giải : Thử thay $\mathrm{x}$ bởi $\mathrm{y}$ thì $\mathrm{P}=\mathrm{y}^2(\mathrm{y}-\mathrm{z})+\mathrm{y}^2(\mathrm{z}-\mathrm{y})=0$. Như vậy $\mathrm{P}$ chia hết cho $\mathrm{x}-\mathrm{y}$.

Ta lại thấy nếu thay $\mathrm{x}$ bởi $\mathrm{y}$, thay $\mathrm{y}$ bởi $\mathrm{z}$, thay $\mathrm{z}$ bởi $\mathrm{x}$ thì $\mathrm{P}$ không đổi ( $\mathrm{ta}$ nói đa thức $\mathrm{P}$ có thể hoán vị vòng quanh $\mathrm{x} \rightarrow \mathrm{y} \rightarrow \mathrm{z} \rightarrow \mathrm{x}$ ). Do đó, nếu $\mathrm{P}$ đã chia hết cho $\mathrm{x}-\mathrm{y}$ thì cũng chia hết cho $\mathrm{y}-\mathrm{z}$ và $\mathrm{z}-\mathrm{x}$. Vậy $\mathrm{P}$ có dạng

$\mathrm{k}(\mathrm{x}-\mathrm{y})(\mathrm{y}-\mathrm{z})(\mathrm{z}-\mathrm{x}) \text {. }$

Ta thấy $\mathrm{k}$ phải là hằng số (không chứa biến) vì $\mathrm{P}$ có bậc ba đối với tập hợp các biến $\mathrm{x}, \mathrm{y}, \mathrm{z}$, còn tích $(\mathrm{x}-\mathrm{y})(\mathrm{y}-\mathrm{z})(\mathrm{z}-\mathrm{x})$ cũng có bậc ba đối với tập hợp các biến $\mathrm{x}, \mathrm{y}, \mathrm{z}$.

Vì đẳng thức $\mathrm{x}^2(\mathrm{y}-\mathrm{z})+\mathrm{y}^2(\mathrm{z}-\mathrm{x})+\mathrm{z}^2(\mathrm{x}-\mathrm{y})=\mathrm{k}(\mathrm{x}-\mathrm{y})(\mathrm{y}-\mathrm{z})(\mathrm{z}-\mathrm{x})$ đúng với mọi $\mathrm{x}, \mathrm{y}, \mathrm{z}$ nên ta gán cho các biến $\mathrm{x}, \mathrm{y}, \mathrm{z}$ các giá trị riêng, chẳng hạn $\mathrm{x}=2$, $\mathrm{y}=1$, $\mathrm{z}=0$(*), ta được :

$4 \cdot 1+1 \cdot(-2)+0=\mathrm{k} \cdot 1 \cdot 1 \cdot(-2) \Leftrightarrow 2=-2 \mathrm{k} \Leftrightarrow \mathrm{k}=-1 \text {. }$

Vậy $P=-(x-y)(y-z)(z-x)=(x-y)(y-z)(x-z)$.

(*) Các giá trị của $\mathrm{x}, \mathrm{y}, \mathrm{z}$ có thể chọn tuỳ ý, chỉ cần chúng đôi một khác nhau để

$(x-y)(y-z)(z-x) \neq 0$

 

BÀI TẬP

 

Phân tích các đa thức sau thành nhân tử (từ bài 1 đến bài 14)

1. a) $6 \mathrm{x}^2-11 \mathrm{x}+3$

b) $2 x^2+3 x-27$

c) $2 x^2-5 x y-3 y^2$

2. a) $x^3+2 x-3$;

b) $x^3-7 x+6$

c) $x^3+5 x^2+8 x+4$

d) $x^3-9 x^2+6 x+16$

e) $x^3-x^2-x-2$;

g) $x^3+x^2-x+2$;

h) $x^3-6 x^2-x+30$.

3. $x^3-7 x-6$ (giải bằng nhiều cách).

4. a) $27 \mathrm{x}^3-27 \mathrm{x}^2+18 \mathrm{x}-4$

b) $2 x^3-x^2+5 x+3$;

c) $\left(x^2-3\right)^2+16$.

5. a) $\left(x^2+x\right)^2-2\left(x^2+x\right)-15$;

b) $x^2+2 x y+y^2-x-y-12$

c) $\left(x^2+x+1\right)\left(x^2+x+2\right)-12$;

d) $(x+2)(x+3)(x+4)(x+5)-24$

6. a) $(x+a)(x+2 a)(x+3 a)(x+4 a)+a^4$;

b) $\left(x^2+y^2+z^2\right)(x+y+z)^2+(x y+y z+z x)^2$;

$\left.c^*\right) 2\left(x^4+y^4+z^4\right)-\left(x^2+y^2+z^2\right)^2-2\left(x^2+y^2+z^2\right)(x+y+z)^2+$

$+(x+y+z)^4$

7*. $(\mathrm{a}+\mathrm{b}+\mathrm{c})^3-4\left(\mathrm{a}^3+\mathrm{b}^3+\mathrm{c}^3\right)-12 \mathrm{abc}$ bằng cách đổi biến : đặt $\mathrm{a}+\mathrm{b}=\mathrm{m}$, $a-b=n$.

8. a) $4 x^4-32 x^2+1$;

b) $x^6+27$;

c) $3\left(x^4+x^2+1\right)-\left(x^2+x+1\right)^2$

d) $\left(2 x^2-4\right)^2+9$

9. a) $4 x^4+1$

b) $4 x^4+y^4$;

c) $x^4+324$.

10. a) $x^5+x^4+1$

b) $x^5+x+1$

c) $x^8+x^7+1$

d) $x^5-x^4-1$

e) $x^7+x^5+1$

g) $x^8+x^4+1$.

11. a) $a^6+a^4+a^2 b^2+b^4-b^6$

$\left.b^*\right) x^3+3 x y+y^3-1$

12. Dùng phương pháp hệ số bất định :

a) $4 x^4+4 x^3+5 x^2+2 x+1$

b) $x^4-7 x^3+14 x^2-7 x+1$

c) $x^4-8 x+63$

d) $(x+1)^4+\left(x^2+x+1\right)^2$.

13* *. a) $x^8+14 x^4+1$;

b) $x^8+98 x^4+1$

14. Dùng phương pháp xét giá trị riêng :

$\mathrm{M}=\mathrm{a}(\mathrm{b}+\mathrm{c}-\mathrm{a})^2+\mathrm{b}(\mathrm{c}+\mathrm{a}-\mathrm{b})^2+\mathrm{c}(\mathrm{a}+\mathrm{b}-\mathrm{c})^2+$

$+(\mathrm{a}+\mathrm{b}-\mathrm{c})(\mathrm{b}+\mathrm{c}-\mathrm{a})(\mathrm{c}+\mathrm{a}-\mathrm{b})$

$180(3)$. Chứng minh rằng tích của bốn số tự nhiên liên tiếp cộng thêm 1 là một số chính phương.

15*. Chứng minh rằng số $\mathrm{A}=(\mathrm{n}+1)^4+\mathrm{n}^4+1$ chia hết cho một số chính phương khác 1 với mọi số n nguyên dương.

16. Tìm các số nguyên a, b, c sao cho khi phân tích đa thức $(x+a)(x-4)-7$ thành nhân tử ta được $(\mathrm{x}+\mathrm{b})(\mathrm{x}+\mathrm{c})$.

17. Tìm các số hữu tỉ $a, b, c$ sao cho khi phân tích đa thức $x^3+a x^2+b x+c$ thành nhân tử ta được $(x+a)(x+b)(x+c)$.

18. Số tự nhiên $\mathrm{n}$ có thể nhận bao nhiêu giá trị, biết rằng khi phân tích đa thức $x^2+x-n$ thành nhân tử ta được $(x-a)(x+b)$ với $a, b$ là các số tự nhiên và $1<\mathrm{n}<100$ ?

19. Cho $A=a^2+b^2+c^2$, trong đó $a$ và $b$ là hai số tự nhiên liên tiếp, $\mathrm{c}=\mathrm{ab}$. Chứng minh rằng $\sqrt{\mathrm{A}}$ là một số tự nhiên lẻ.

 

 

 

 

 

 

 

 

 

 

 

 

 

 

 

 

 

PHÂN THỨC ĐẠI SỐ – P.1

TÍNH CHẤT CƠ BẢN CỦA PHÂN THỨC.

RÚT GỌN PHÂN THỨC

Phân thức đại số là một biểu thức có dạng $\frac{\mathrm{A}}{\mathrm{B}}$, trong đó $\mathrm{A}$ và $\mathrm{B}$ là các đa thức, $\mathrm{B} \neq 0$.

Phân thức đại số có các tính chất cơ bản sau :

$-$ Nếu nhân cả tử thức và mẫu thức của một phân thức với cùng một đa thức khác 0 thì được một phân thức bằng phân thức đã cho.

$-$ Nếu chia cả tử thức và mầu thức của một phân thức cho cùng một nhân tử chung của chúng thì được một phân thức bằng phân thức đã cho.

Muốn rút gọn một phân thức đại số, ta có thể :

$-$ Phân tích tử thức và mẫu thức thành nhân tử ;

$-$ Chia cả tử thức và mẫu thức cho nhân tử chung.

Ví dụ 1. Cho phân thức

$M=\frac{\left(a^2+b^2+c^2\right)(a+b+c)^2+(a b+b c+c a)^2}{(a+b+c)^2-(a b+b c+c a)}$

a) Tìm các giá trị của $\mathrm{a}, \mathrm{b}, \mathrm{c}$ để phân thức được xác định (tức là để mẫu . khác 0).

b) Rút gọn phân thức $M$.

Giải : Ta có

$(a+b+c)^2-(a b+b c+c a)=0 \Leftrightarrow a^2+b^2+c^2+a b+b c+c a=0 $

$\Leftrightarrow  2 a^2+2 b^2+2 c^2+2 a b+2 b c+2 c a=0 $

$\Leftrightarrow (a+b)^2+(b+c)^2+(c+a)^2=0 \Leftrightarrow a+b=b+c=c+a=0 $

$\Leftrightarrow  a=b=c=0$

Vậy điều kiện để phân thức $\mathrm{M}$ được xác định là $\mathrm{a}, \mathrm{b}$, $\mathrm{c}$ không đồng thời bằng $0 .$

b) Chú ý rằng $(a+b+c)^2=a^2+b^2+c^2+2(a b+b c+c a)$. Do đó, ta đặt $a^2+b^2+c^2=x, a b+b c+c a=y$. Khi đó $(a+b+c)^2=x+2 y$. Ta có

$M=\frac{x(x+2 y)+y^2}{x+2 y-y}=\frac{x^2+2 x y+y^2}{x+y}=\frac{(x+y)^2}{x+y}=x+y$

$=a^2+b^2+c^2+a b+b c+c a .$

Ví  dụ 2.Rút gọn phân thức

$A=\frac{(b-c)^3+(c-a)^3+(a-b)^3}{a^2(b-c)+b^2(c-a)+c^2(a-b)} .$

Giải : Phân tích mẫu thức thành nhân tử :

$a^2(b-c)+b^2(c-a)+c^2(a-b)=a^2(b-c)+b^2 c-a b^2+a c^2-b c^2 $

$= a^2(b-c)+b c(b-c)-a\left(b^2-c^2\right)=(b-c)\left(a a^2+b c-a b-a c\right) $

$=(b-c)[a(a-b)-c(a-b)]=(b-c)(a-b)(a-c) . $

$\text { Do đó } \quad A=\frac{(b-c)^3+(c-a)^3+(a-b)^3}{-(a-b)(b-c)(c-a)} .$

Ta có nhận xét : Nếu $x+y+z=0$ thì $x^3+y^3+z^3=3 x y z$ (chứng minh : xem bài tập 42). Đặt $b-c=x, c-a=y, a-b=z$ thì $x+y+z=0$. Theo nhận xét trên :

$A=\frac{x^3+y^3+z^3}{-x y z}=\frac{3 x y z}{-x y z}=-3$

Ví dụ 3. Chứng minh rằng với mọi số nguyên n thì phân số $\frac{n^3+2 n}{n^4+3 n^2+1}$ là phân số tối giản.

Giải : Để chứng minh phân số đã cho là tối giản, ta sẽ chứng tỏ rằng tử và mẫu chỉ có ước chung là $\pm 1$.

Gọi d là ước chung của $n^3+2 n$ và $n^4+3 n^2+1$. Ta có :

$n^3+2 n \vdots d \Rightarrow n\left(n^3+2 n\right) \vdots d \Rightarrow n^4+2 n^2 \vdots d $

$n^4+3 n^2+1-\left(n^4+2 n^2\right)=n^2+1 \vdots d \Rightarrow\left(n^2+1\right)^2=n^4+2 n^2+1 \vdots d$

Từ $(1)$ và $(2)$ suy ra

$\left(n^4+2 n^2+1\right)-\left(n^4+2 n^2\right): d \Rightarrow 1: d \Rightarrow d=\pm 1 .$

Vậy $\frac{n^3+2 n}{n^4+3 n^2+1}$ là phân số tối giản.

Ví dụ 4. Chứng minh rằng

$1+x+x^2+x^3+\ldots+x^{31}=(1+x)\left(1+x^2\right)\left(1+x^4\right)\left(1+x^8\right)\left(1+x^{16}\right)\quad(1)$

Giải : Gọi vế trái của đẳng thức (1) là $\mathrm{A}$, vế phải là $\mathrm{B}$.

Ta có $(1-\mathrm{x}) \cdot \mathrm{A}=1-\mathrm{x}^{32}$ theo hằng đẳng thức 8 ,

$(1-x) \cdot B=(1-x)(1+x)\left(1+x^2\right)\left(1+x^4\right)\left(1+x^8\right)\left(1+x^{16}\right)=1-x^{32} \text {. }$

Nếu $\mathrm{x} \neq 1$ thì $\mathrm{A}$ và $\mathrm{B}$ đều bằng phân thức $\frac{1-\mathrm{x}^{32}}{1-\mathrm{x}}$. Do đó $\mathrm{A}=\mathrm{B}$.

Nếu $\mathrm{x}=1$ thì hai vế của (1) đều bằng 32 . Do đó $\mathrm{A}=\mathrm{B}$.

Trong cả hai trường hợp, đẳng thức (1) đều đúng.

 

BÀI TẬP

1. Tìm giá trị của $\mathrm{x}$ để các phân thức sau bằng 0 :

a) $\frac{x^4+x^3+x+1}{x^4-x^3+2 x^2-x+1}$

b) $\frac{x^4-5 x^2+4}{x^4-10 x^2+9}$

2. Rút gọn các phân thức :

a) $\mathrm{A}=\frac{1235.2469-1234}{1234.2469+1235}$;

b) $\mathrm{B}=\frac{4002}{1000.1002-999.1001}$.

3. Rút gọn các phân thức :

a) $\frac{3 x^3-7 x^2+5 x-1}{2 x^3-x^2-4 x+3}$

b) $\frac{(x-y)^3-3 x y(x+y)+y^3}{x-6 y}$

c) $\frac{x^2+y^2+z^2-2 x y+2 x z-2 y z}{x^2-2 x y+y^2-z^2}$.

4. Rút gọn các phân thức với n là số tự nhiên :

a) $\frac{(n+1) !}{n !(n+2)}$

b) $\frac{n !}{(n+1) !-n !}$

c) $\frac{(n+1) !-(n+2) !}{(n+1) !+(n+2) !}$

5. Rút gọn các phân thức :

a) $\frac{a^2(b-c)+b^2(c-a)+c^2(a-b)}{a b^2-a c^2-b^3+b c^2}$;

b) $\frac{2 x^3-7 x^2-12 x+45}{3 x^3-19 x^2+33 x-9}$

c) $\frac{x^3-y^3+z^3+3 x y z}{(x+y)^2+(y+z)^2+(z-x)^2}$

d) $\frac{x^3+y^3+z^3-3 x y z}{(x-y)^2+(y-z)^2+(z-x)^2}$.

6. Chứng minh rằng các phân số sau tối giản với mọi số tự nhiên $\mathrm{n}$ :

a) $\frac{3 n+1}{5 n+2}$;

b) $\frac{12 n+1}{30 n+2}$

$\left.c^*\right) \frac{n^3+2 n}{n^4+3 n^2+1}$

d) $\frac{2 n+1}{2 n^2-1}$.

7. Chứng minh rằng phân số $\frac{n^7+n^2+1}{n^8+n+1}$ không tối giản với mọi số nguyên dương $n$.

8. Viết gọn biểu thức sau dưới dạng một phân thức :

$\left(x^2-x+1\right)\left(x^4-x^2+1\right)\left(x^8-x^4+1\right)\left(x^{16}-x^8+1\right)\left(x^{32}-x^{16}+1\right)$

9. Cho biết $\mathrm{x}, \mathrm{y}, \mathrm{z}$ khác 0 và $\frac{(\mathrm{ax}+\mathrm{by}+\mathrm{cz})^2}{\mathrm{x}^2+\mathrm{y}^2+\mathrm{z}^2}=\mathrm{a}^2+\mathrm{b}^2+\mathrm{c}^2$.

Chứng minh rằng $\frac{\mathrm{a}}{\mathrm{x}}=\frac{\mathrm{b}}{\mathrm{y}}=\frac{\mathrm{c}}{\mathrm{z}}$.

10*. Cho biết $\mathrm{ax}+\mathrm{by}+\mathrm{cz}=0$.

Rút gọn $\mathrm{A}=\frac{\mathrm{bc}(\mathrm{y}-\mathrm{z})^2+\mathrm{ca}(\mathrm{z}-\mathrm{x})^2+\mathrm{ab}(\mathrm{x}-\mathrm{y})^2}{a \mathrm{x}^2+\mathrm{by}^2+c \mathrm{z}^2}$.

11. Rút gọn $\frac{\mathrm{x}^2+\mathrm{y}^2+\mathrm{z}^2}{(\mathrm{y}-\mathrm{z})^2+(\mathrm{z}-\mathrm{x})^2+(\mathrm{x}-\mathrm{y})^2}$, biết rằng $\mathrm{x}+\mathrm{y}+\mathrm{z}=0$.

12. Tính giá trị của biểu thức $\mathrm{A}=\frac{\mathrm{x}-\mathrm{y}}{\mathrm{x}+\mathrm{y}}$, biết $\mathrm{x}^2-2 \mathrm{y}^2=\mathrm{xy}(\mathrm{y} \neq 0 ; \mathrm{x}+\mathrm{y} \neq 0)$.

13. Tính giá trị của phân thức $A=\frac{3 x-2 y}{3 x+2 y}$, biết rằng $9 x^2+4 y^2=20 x y$ và $2 y<3 x<0$

14. Cho $3 \mathrm{x}-\mathrm{y}=3 \mathrm{z}$ và $2 \mathrm{x}+\mathrm{y}=7 \mathrm{z}$. Tính giá trị của biểu thức

$M=\frac{x^2-2 x y}{x^2+y^2}(x \neq 0, y \neq 0)$

15. Tìm số nguyên $x$ để phân thức sau có giá trị là số nguyên :

a) $\frac{3}{2 x-1}$

b) $\frac{5}{x^2+1}$;

c) $\frac{7}{x^2-x+1}$

d) $\frac{x^2-59}{x+8}$

e) $\frac{x+2}{x^2+4}$

16. Tìm số hữu tỉ $x$ để phân thức $\frac{10}{x^2+1}$ có giá trị là số nguyên.

17*. Chứng minh rằng nếu các chữ số $\mathrm{a}, \mathrm{b}, \mathrm{c}$ khác 0 thoả mãn điều kiện $\overline{\mathrm{ab}}: \overline{\mathrm{bc}}=\mathrm{a}: \mathrm{c}$ thì $\overline{\mathrm{abbb}}: \overline{\mathrm{bbbc}}=\mathrm{a}: \mathrm{c} .$

 

 

 

 

 

 

 

 

 

 

 

 

 

 

 

 

 

 

 

 

 

 

 

 

 

 

 

 

 

 

 

 

 

 

 

 

 

 

 

 

 

 

 

 

 

 

 

 

 

 

 

 

 

 

 

PHÉP NHÂN VÀ PHÉP CHIA ĐA THỨC – P.2

CÁC HẰNG ĐẲNG THỨC ĐÁNG NHỚ

 

Thực hiện phép nhân đa thức, ta được các hằng đẳng thức sau :

1. $(a+b)^2=a^2+2 a b+b^2$.

2. $(a-b)^2=a^2-2 a b+b^2$.

3. $(a+b)(a-b)=a^2-b^2$.

4. $(a+b)^3=a^3+3 a^2 b+3 a b^2+b^3$

$(a+b)^3=a^3+b^3+3 a b(a+b) \text {. }$

5. $(a-b)^3=a^3-3 a^2 b+3 a b^2-b^3$

$(a-b)^3=a^3-b^3-3 a b(a-b)$

6. $(a+b)\left(a^2-a b+b^2\right)=a^3+b^3$

7. $(a-b)\left(a^2+a b+b^2\right)=a^3-b^3$.

Ta cũng có :

$(a+b+c)^2=a^2+b^2+c^2+2 a b+2 a c+2 b c .$

Tổng quát của các hằng đẳng thức 3 và 7 , ta có hằng đẳng thức :

8. $a^n-b^n=(a-b)\left(a^{n-1}+a^{n-2} b+a^{n-3} b^2+\ldots+a b^{n-2}+b^{n-1}\right)$

với mọi số nguyên dương $\mathrm{n}$.

Tổng quát của hằng đẳng thức 6 , ta có hằng đẳng thức :

9. $a^n+b^n=(a+b)\left(a^{n-1}-a^{n-2} b+a^{n-3} b^2-\ldots-a b^{n-2}+b^{n-1}\right)$

với mọi số lẻ n.

Tổng quát của các hằng đẳng thức $1,2,4,5$, ta có công thức Niu-tơn (xem chuyên đề Tính chia hết đối với số nguyên).

Ví dụ 1. Chứng minh rằng số 3599 viết được dưới dạng tích của hai số tự nhiên khác 1 .

Giải : $\quad 3599=3600-1=60^2-1=(60+1)(60-1)=61.59$.

Ví dụ 2. Chứng minh rằng biểu thức sau viết được dưới dạng tổng các bình phương của hai biểu thức :

$x^2+2(x+1)^2+3(x+2)^2+4(x+3)^2$

Giải: $\mathrm{x}^2+2(\mathrm{x}+1)^2+3(\mathrm{x}+2)^2+4(\mathrm{x}+3)^2=$

$=x^2+2\left(x^2+2 x+1\right)+3\left(x^2+4 x+4\right)+4\left(x^2+6 x+9\right) $

$=x^2+2 x^2+4 x+2+3 x^2+12 x+12+4 x^2+24 x+36 $

$=10 x^2+40 x+50 $

$=\left(x^2+10 x+25\right)+\left(9 x^2+30 x+25\right) $

$=(x+5)^2+(3 x+5)^2$

Ví dụ 3. Cho

$x+y+z=0 $

$4x y+y z+z x=0$

Chứng minh rằng $\mathrm{x}=\mathrm{y}=\mathrm{z}$.

Giải : Ta có $(\mathrm{x}+\mathrm{y}+\mathrm{z})^2=\mathrm{x}^2+\mathrm{y}^2+\mathrm{z}^2+2(\mathrm{xy}+\mathrm{yz}+\mathrm{zx})$.

Suy ra

$0=x^2+y^2+z^2+2.0$

hay

$\text { Vậy } x=y=z(=0) \text {. }$

Ví dụ 4 :

a) Tính $A=-1^2+2^2-3^2+4^2-\ldots-99^2+100^2$.

b) Tính $\mathrm{A}=-1^2+2^2-3^2+4^2-\ldots+(-1)^{\mathrm{n}} \cdot \mathrm{n}^2$.

Giải: a) $\mathrm{A}=\left(2^2-1^2\right)+\left(4^2-3^2\right)+\ldots+\left(100^2-99^2\right)$

$=(2-1)(1+2)+(4-3)(3+4)+\ldots+(100-99)(99+100) $

$=1+2+3+4+\ldots+99+100 $

$=\frac{100.101}{2}=5050 .$

b) Xét hai trường hợp :

Nếu n chẵn thì $\mathrm{A}=\left(2^2-1^2\right)+\left(4^2-3^2\right)+\ldots+\left[\mathrm{n}^2-(\mathrm{n}-1)^2\right]$

$=1+2+3+4+\ldots+(n-1)+n$

$=\frac{\mathrm{n}(\mathrm{n}+1)}{2} \text {. }$

Nếu n lẻ thì $\mathrm{A}=\left(2^2-1^2\right)+\left(4^2-3^2\right)+\ldots+\left[(\mathrm{n}-1)^2-(\mathrm{n}-2)^2\right]-\mathrm{n}^2$

$=1+2+3+4+\ldots+(n-1)-n^2 $

$=\frac{n(n-1)}{2}-n^2=-\frac{n(n+1)}{2}$

Chú ý : Hai kết quả trên có thể viết chung trong một công thức

$(-1)^{\mathrm{n}} \cdot \frac{\mathrm{n}(\mathrm{n}+1)}{2}$

Ví dụ 5. Cho

$x+y=a+b\quad(1)$

$x^2+y^2=a^2+b^2\quad(2)$

Chứng minh rằng $x^3+y^3=a^3+b^3$.

Giải : Ta có : $\quad \mathrm{x}^3+\mathrm{y}^3=(\mathrm{x}+\mathrm{y})\left(\mathrm{x}^2-\mathrm{xy}+\mathrm{y}^2\right)\quad(3)$.

Từ (1) suy ra : $\quad(x+y)^2=(a+b)^2$,

tức là $\quad x^2+2 x y+y^2=a^2+2 a b+b^2$.

Do $x^2+y^2=a^2+b^2$ nên $2 x y=2 a b$, suy ra $x y=a b\quad(4)$

Thay các kết quả (1), (2), (4) vào (3), ta được

$x^3+y^3=(x+y)\left(x^2+y^2-x y\right)=(a+b)\left(a^2+b^2-a b\right)=a^3+b^3 .$

Ví dụ 6. Cho $a+b=m, a-b=n$. Tính $a b$ và $a^3-b^3$ theo $m$ và $n$.

Giải :

Cách 1. Từ $\mathrm{a}+\mathrm{b}=\mathrm{m}, \mathrm{a}-\mathrm{b}=\mathrm{n}$, ta tính được $\mathrm{b}=\frac{\mathrm{m}-\mathrm{n}}{2}, \mathrm{a}=\frac{\mathrm{m}+\mathrm{n}}{2}$.

Do đó $\quad \mathrm{ab}=\frac{\mathrm{m}+\mathrm{n}}{2} \cdot \frac{\mathrm{m}-\mathrm{n}}{2}=\frac{\mathrm{m}^2-\mathrm{n}^2}{4} ;$

$a^3-b^3=\left(\frac{m+n}{2}\right)^3-\left(\frac{m-n}{2}\right)^3=\frac{(m+n)^3-(m-n)^3}{8}$

Rút gọn biểu thức trên, ta được $\frac{3 \mathrm{~m}^2 \mathrm{n}+\mathrm{n}^3}{4}$.

Cách 2. Ta có

$4 a b =(a+b)^2-(a-b)^2=m^2-n^2 \text { nên } a b=\frac{m^2-n^2}{4} . $

$\text { Ta có } a^3-b^3 =(a-b)\left(a^2+a b+b^2\right)=(a-b)\left[(a+b)^2-a b\right] $

$=n\left(m^2-\frac{m^2-n^2}{4}\right)=\frac{n\left(3 m^2+n^2\right)}{4}=\frac{3 m^2 n+n^3}{4} .$

BÀI TẬP

16. Tính giá trị của các biểu thức :

a) $\frac{63^2-47^2}{215^2-105^2}$

b) $\frac{437^2-363^2}{537^2-463^2}$

17. So sánh $\mathrm{A}=26^2-24^2$ và $\mathrm{B}=27^2-25^2$.

18. Tìm $\mathrm{x}$, biết :

$4(x+1)^2+(2 x-1)^2-8(x-1)(x+1)=11$

19. Rút gọn các biểu thức :

a) $2 x(2 x-1)^2-3 x(x+3)(x-3)-4 x(x+1)^2$;

b) $(a-b+c)^2-(b-c)^2+2 a b-2 a c$;

c) $(3 x+1)^2-2(3 x+1)(3 x+5)+(3 x+5)^2$;

d) $(3+1)\left(3^2+1\right)\left(3^4+1\right)\left(3^8+1\right)\left(3^{16}+1\right)\left(3^{32}+1\right)$;

e) $(a+b-c)^2+(a-b+c)^2-2(b-c)^2$

g) $(a+b+c)^2+(a-b-c)^2+(b-c-a)^2+(c-a-b)^2$;

h) $(a+b+c+d)^2+(a+b-c-d)^2+(a+c-b-d)^2+(a+d-b-c)^2$.

20. Cho $x+y=3$. Tính giá trị của biểu thức

$A=x^2+2 x y+y^2-4 x-4 y+1 $

21. Cho $\mathrm{a}^2+\mathrm{b}^2+\mathrm{c}^2=\mathrm{m}$. Tính giá trị của biểu thức sau theo $\mathrm{m}$ :

$A=(2 a+2 b-c)^2+(2 b+2 c-a)^2+(2 c+2 a-b)^2 .$

22. Hãy viết các số sau đây dưới dạng tích của hai số tự nhiên khác 1 :

a) $899$

b) $9991$

23. Chứng minh rằng hiệu sau đây là một số gồm toàn các chữ số như nhau :

$7778^2-2223^2$

24. Chứng minh các hằng đẳng thức :

a) $(a+b+c)^2+a^2+b^2+c^2=(a+b)^2+(b+c)^2+(c+a)^2$

b) $x^4+y^4+(x+y)^4=2\left(x^2+x y+y^2\right)^2$

25. Cho $\mathrm{a}^2-\mathrm{b}^2=4 \mathrm{c}^2$. Chứng minh hằng đẳng thức

$(5 a-3 b+8 c)(5 a-3 b-8 c)=(3 a-5 b)^2$

26. Chứng minh rằng nếu $\left(a^2+b^2\right)\left(x^2+y^2\right)=(a x+b y)^2$ với $x, y$ khác 0 thì $\frac{\mathrm{a}}{\mathrm{x}}=\frac{\mathrm{b}}{\mathrm{y}}$

27. Chứng minh rằng nếu $\left(\mathrm{a}^2+\mathrm{b}^2+\mathrm{c}^2\right)\left(\mathrm{x}^2+\mathrm{y}^2+\mathrm{z}^2\right)=(\mathrm{ax}+\mathrm{by}+\mathrm{cz})^2$ với $x, y, z$ khác 0 thì $\frac{a}{x}=\frac{b}{y}=\frac{c}{z}$.

28. Cho $(a+b)^2=2\left(a^2+b^2\right)$. Chứng minh rằng $a=b$.

29. Chứng minh rằng $\mathrm{a}=\mathrm{b}=\mathrm{c}$ nếu có một trong các điều kiện sau :

a) $a^2+b^2+c^2=a b+b c+c a$

b) $(a+b+c)^2=3\left(a^2+b^2+c^2\right)$

c) $(a+b+c)^2=3(a b+b c+c a)$.

  1. Hãy viết các biểu thức sau dưới dạng tổng của ba bình phương :

a) $(a+b+c)^2+a^2+b^2+c^2$

b) $2(a-b)(c-b)+2(b-a)(c-a)+2(b-c)(a-c)$

31. Tính giá trị của biểu thức $\mathrm{a}^4+\mathrm{b}^4+\mathrm{c}^4$, biết rằng $\mathrm{a}+\mathrm{b}+\mathrm{c}=0$ và :

a) $a^2+b^2+c^2=2$;

b) $a^2+b^2+c^2=1$.

32. Cho $\mathrm{a}+\mathrm{b}+\mathrm{c}=0$. Chứng minh $\mathrm{a}^4+\mathrm{b}^4+\mathrm{c}^4$ bằng mỗi biểu thức :

a) $2\left(a^2 b^2+b^2 c^2+c^2 a^2\right)$;

b) $2(a b+b c+c a)^2$

c) $\frac{\left(a^2+b^2+c^2\right)^2}{2}$

33. Chứng minh rằng các biểu thức sau luôn luôn có giá trị dương với mọi giá trị của biến :

a) $9 x^2-6 x+2$

b) $\mathrm{x}^2+\mathrm{x}+1$

c) $2 x^2+2 x+1$.

34. Tìm giá trị nhỏ nhất của các biểu thức :

a) $A=x^2-3 x+5 ;$

b) $B=(2 x-1)^2+(x+2)^2$

35. Tìm giá trị lớn nhất của các biểu thức :

a) $A=4-x^2+2 x$

b) $B=4 x-x^2$

36. Chứng minh rằng :

a) Nếu $\mathrm{p}$ và $\mathrm{p}^2+8$ là các số nguyên tố thì $\mathrm{p}^2+2$ cũng là số nguyên tố.

b) Nếu $\mathrm{p}$ và $8 \mathrm{p}^2+1$ là các số nguyên tố thì $2 \mathrm{p}+1$ cũng là số nguyên tố.

37. Chứng minh rằng các số sau là hợp số :

a) 999991 ;

b) 1000027 .

38. Thực hiện phép tính :

a) $(x-2)^3-x(x+1)(x-1)+6 x(x-3)$

b) $(x-2)\left(x^2-2 x+4\right)(x+2)\left(x^2+2 x+4\right)$.

39. Tìm $x$, biết :

a) $(x-3)\left(x^2+3 x+9\right)+x(x+2)(2-x)=1$

b) $(x+1)^3-(x-1)^3-6(x-1)^2=-10$

40. Rút gọn các biểu thức :

a) $(a+b+c)^3-(b+c-a)^3-(a+c-b)^3-(a+b-c)^3$

b) $(a+b)^3+(b+c)^3+(c+a)^3-3(a+b)(b+c)(c+a)$

41. Chứng minh các hằng đẳng thức :

a) $(a+b+c)^3-a^3-b^3-c^3=3(a+b)(b+c)(c+a)$.

b) $a^3+b^3+c^3-3 a b c=(a+b+c)\left(a^2+b^2+c^2-a b-b c-c a\right)$.

42. Cho $a+b+c=0$. Chứng minh rằng $a^3+b^3+c^3=3 a b c$.

43. Cho $\mathrm{x}+\mathrm{y}=\mathrm{a}$ và $\mathrm{xy}=\mathrm{b}$. Tính giá trị của các biểu thức sau theo $\mathrm{a}$ và $\mathrm{b}$ :

a) $x^2+y^2$

b) $x^3+y^3$

c) $x^4+y^4$;

d) $x^5+y^5$.

44. a) Cho $x+y=1$. Tính giá trị của biểu thức $x^3+y^3+3 x y$.

b) Cho $\mathrm{x}-\mathrm{y}=1$. Tính giá trị của biểu thức $\mathrm{x}^3-\mathrm{y}^3-3 \mathrm{xy}$.

45. Cho $\mathrm{a}+\mathrm{b}=1$. Tính giá trị của biểu thức

$M=a^3+b^3+3 a b\left(a^2+b^2\right)+6 a^2 b^2(a+b)$

46. a) Cho $x+y=2$ và $x^2+y^2=10$. Tính giá trị của biểu thức $x^3+y^3$.

b) Cho $x+y=a$ và $x^2+y^2=b$. Tính $x^3+y^3$ theo a và $b$.

47. Chứng minh rằng :

a) Nếu số n’ là tổng của hai số chính phương thì 2 n cũng là tổng của hai số chính phương.

b) Nếu số $2 \mathrm{n}$ là tổng của hai số chính phương thì n cũng là tổng của hai số chính phương.

c) Nếu số $\mathrm{n}$ là tổng của hai số chính phương thì $\mathrm{n}^2$ cũng là tổng của hai số chính phương.

d) Nếu mỗi số m và $\mathrm{n}$ đều là tổng của hai số chính phương thì tích mn cũng là tổng của hai số chính phương.

48. Chứng minh rằng với mọi số tự nhiên $\mathrm{a}$, tồn tại số tự nhiên $\mathrm{b}$ sao cho $\mathrm{ab}+4$ là số chính phương.

49. Cho a là số gồm $2 \mathrm{n}$ chữ số $1, \mathrm{~b}$ là số gồm $\mathrm{n}+1$ chữ số $1, \mathrm{c}$ là số gồm $\mathrm{n}$ chữ số 6. Chứng minh rằng $\mathrm{a}+\mathrm{b}+\mathrm{c}+8$ là số chính phương.

50. Chứng minh rằng biểu thức sau không là lập phương của một số tự nhiên :

$10^{150}+5.10^{50}+1 .$

51. Chứng minh rằng tích ba số nguyền dương liên tiếp không là lập phương của một số tự nhiên.

52. Chia 27 quả cân có khối lượng $10,20,30, \ldots, 270$ gam thành ba nhóm có khối lượng bằng nhau.

53*. Chia 18 quả cân có khối lượng $1^2, 2^2, 3^2, \ldots, 18^2$ gam thành ba nhóm có khối lượng bằng nhau.

54*. Chia 27 quả cân có khối lượng $1^2, 2^2, 3^2, \ldots, 27^2$ gam thành ba nhóm có khối lượng bằng nhau.

PHÉP NHÂN VÀ PHÉP CHIA ĐA THỨC – P.1

NHÂN ĐA THỨC

 

Ví dụ 1. Tính giá trị của biểu thức

$A=x^4-17 x^3+17 x^2-17 x+20 \text { tại } x=16 \text {. }$

Giải : Cách 1. Chú ý rằng $\mathrm{x}=16$ nên $\mathrm{x}-16=0$, do đó ta biến đổi để biểu thức $\mathrm{A}$ chứa nhiều biểu thức dạng $\mathrm{x}-16$.

$A =x^4-16 x^3-x^3+16 x^2+x^2-16 x-x+16+4 $

$=x^3(x-16)-x^2(x-16)+x(x-16)-(x-16)+4=4$

Cách 2. Trong biểu thức $\mathrm{A}$, ta thay các số 17 bởi $\mathrm{x}+1$, còn 20 thay bởi $\mathrm{x}+4$.

$A =x^4-x^3(x+1)+x^2(x+1)-x(x+1)+x+4$

$=x^4-x^4-x^3+x^3+x^2-x^2-x+x+4=4$

Ví dụ 2. Tìm ba số tự nhiên liên tiếp, biết rằng nếu cộng ba tích của hai trong ba số ấy, ta được 242 .

Giải : Gọi $\mathrm{x}-1, \mathrm{x}, \mathrm{x}+1$ là ba số tự nhiên liên tiếp. Ta có :

$x(x-1)+x(x+1)+(x-1)(x+1)=242$

Sau khi rút gọn ta được $3 x^2-1=242$ nên $x^2=81$.

Do $\mathrm{x}$ là số tự nhiên nên $\mathrm{x}=9 .$ Ba số tự nhiên phải tìm là $8 ; 9 ; 10$.

BÀI TẬP

– Nhân đơn thức với đa thức

1. Thực hiện phép tính :

a) $3 x^n\left(6 x^{n-3}+1\right)-2 x^n\left(9 x^{n-3}-1\right)$;

b) $5^{\mathrm{n}+1}-4.5^{\mathrm{n}}$

c) $6^2 \cdot 6^4-4^3\left(3^6-1\right)$

2. Tìm $\mathrm{x}$, biết :

a) $4(18-5 x)-12(3 x-7)=15(2 x-16)-6(x+14)$;

b) $5(3 x+5)-4(2 x-3)=5 x+3(2 x+12)+1$;

c) $2(5 x-8)-3(4 x-5)=4(3 x-4)+11$;

d) $5 x-3{4 x-2[4 x-3(5 x-2)]}=182$.

3. Tính giá trị của các biểu thức :

a) $A=x^3-30 x^2-31 x+1$ tại $x=31$

b) $\mathrm{B}=\mathrm{x}^5-15 \mathrm{x}^4+16 \mathrm{x}^3-29 \mathrm{x}^2+13 \mathrm{x}$ tại $\mathrm{x}=14$

c) $C=x^{14}-10 x^{13}+10 x^{12}-10 x^{11}+\ldots+10 x^2-10 x+10$ tại $x=9$.

4. Tính giá trị của biểu thức sau bằng cách thay số bởi chữ một cách hợp lí :

$A=2 \frac{1}{315} \cdot \frac{1}{651}-\frac{1}{105} \cdot 3 \frac{650}{651}-\frac{4}{315.651}+\frac{4}{105}$

– Nhân đa thức với đa thức

5. Thực hiện phép tính :

a) $(x-1)\left(x^5+x^4+x^3+x^2+x+1\right)$

b) $(x+1)\left(x^6-x^5+x^4-x^3+x^2-x+1\right)$.

6. Tìm x, biết :

a) $(x+2)(x+3)-(x-2)(x+5)=6$;

b) $(3 x+2)(2 x+9)-(x+2)(6 x+1)=(x+1)-(x-6)$;

c) $3(2 x-1)(3 x-1)-(2 x-3)(9 x-1)=0$.

7. Cho $a+b+c=0$. Chứng minh rằng $M=N=P$ với : $\mathrm{M}=\mathrm{a}(\mathrm{a}+\mathrm{b})(\mathrm{a}+\mathrm{c}) ; \quad \mathrm{N}=\mathrm{b}(\mathrm{b}+\mathrm{c})(\mathrm{b}+\mathrm{a}) ; \quad \mathrm{P}=\mathrm{c}(\mathrm{c}+\mathrm{a})(\mathrm{c}+\mathrm{b})$

8. Chứng minh các hằng đẳng thức :

a) $(x+a)(x+b)=x^2+(a+b) x+a b$

b) $(x+a)(x+b)(x+c)=x^3+(a+b+c) x^2+(a b+b c+c a) x+a b c .$

9. Cho $\mathrm{a}+\mathrm{b}+\mathrm{c}=2 \mathrm{p}$. Chứng minh hằng đẳng thức :

$2 b c+b^2+c^2-a^2=4 p(p-a)$

10. Xét các ví dụ : $53.57=3021, \quad 72.78=5616$.

Hãy xây dựng quy tắc nhân nhẩm hai số có hai chữ số, trong đó các chữ số hàng chục bằng nhau, còn các chữ số hàng đơn vị có tổng bằng $10 .$

11. Cho biểu thức

$M=(x-a)(x-b)+(x-b)(x-c)+(x-c)(x-a)+x^2$

Tính $\mathrm{M}$ theo $\mathrm{a}, \mathrm{b}, \mathrm{c}$, biết rằng $\mathrm{x}=\frac{1}{2} \mathrm{a}+\frac{1}{2} \mathrm{~b}+\frac{1}{2} \mathrm{c}$.

12. Cho dãy số $1,3,6,10,15, \ldots, \frac{n(n+1)}{2}, \ldots$

Chứng minh rằng tổng hai số hạng liên tiếp của dãy bao giờ cũng là số chính phương.

13. Số a gồm 31 chữ số 1 , số $\mathrm{b}$ gồm 38 chữ số 1 . Chứng minh rằng $\mathrm{ab}-2$ chia hết cho 3 .

$14^*$ . Số $3^{50}+1$ có là tích của hai số tự nhiên liên tiếp khônğ ?

15*. a) Thực hiện phép tính :

$A=\left(2^9+2^7+1\right)\left(2^{23}-2^{21}+2^{19}-2^{17}+2^{14}-2^{10}+2^9-2^7+1\right)$

b) Số $2^{32}+1$ có là số nguyên tố không ?

 

 

 

 

 

 

 

 

 

 

 

 

 

 

 

 

 

 

 

 

 

 

 

 

 

 

 

 

 

 

 

 

 

 

 

 

 

 

 

 

PHÉP NHÂN VÀ PHÉP CHIA ĐA THỨC – P.4

CHIA ĐA THỨC

 

Đa thức $\mathrm{A}(\mathrm{x})$ gọi là chia hết cho đa thức $\mathrm{B}(\mathrm{x})$ khác 0 nếu tồn tại đa thức $\mathrm{Q}(\mathrm{x})$ sao cho $\mathrm{A}(\mathrm{x})=\mathrm{B}(\mathrm{x}) \cdot \mathrm{Q}(\mathrm{x})$.

Người ta chứng minh được rằng : Với mọi cặp đa thức $\mathrm{A}(\mathrm{x})$ và $\mathrm{B}(\mathrm{x})$ trong đó $\mathrm{B}(\mathrm{x}) \neq 0$, tồn tại duy nhất cặp đa thức $\mathrm{Q}(\mathrm{x})$ và $\mathrm{R}(\mathrm{x})$ sao cho $\mathrm{A}(\mathrm{x})=\mathrm{B}(\mathrm{x}) \cdot \mathrm{Q}(\mathrm{x})+\mathrm{R}(\mathrm{x})$, trong đó $R(x)=0$ hoặc bậc của $R(x)$ nhỏ hơn bậc của $B(x)$.

Nếu $R(x)=0$ thì $A(x)$ chia hết cho $B(x)$. Nếu $R(x) \neq 0$ thì $A(x)$ không chia hết cho $B(x)$, khi đó $Q(x)$ là thương và $R(x)$ là dư của phép chia $A(x)$ cho $B(x)$.

Ví dụ 1. Tìm số tự nhiên $\mathrm{n}$ để đa thức $\mathrm{A}$ chia hết cho đơn thức $\mathrm{B}$ :

$A=3 x^{n-1} y^6-5 x^{n+1} y^4 ; B=2 x^3 y^n$

Tìm thương $\mathrm{A}: \mathrm{B}$ trong trường hợp đó.

Giải : Điều kiện để $\mathrm{A}$ chia hết cho $\mathrm{B}$ là :

$\left\{\begin{array}{r}\mathrm{n}-1 \geq 3 \\ \mathrm{n}+1 \geq 3 \\ 6 \geq \mathrm{n} \\ 4 \geq \mathrm{n}\end{array} \Leftrightarrow\left\{\begin{array}{l}\mathrm{n} \geq 4 \\ \mathrm{n} \leq 4\end{array} \Leftrightarrow \mathrm{n}=4\right.\right.$

Vậy với $\mathrm{n}=4$ thì đa thức $\mathrm{A}$ chia hết cho đơn thức $\mathrm{B}$. Khi đó

$A: B=\left(3 x^3 y^6-5 x^5 y^4\right):\left(2 x^3 y^4\right)=\frac{3}{2} y^2-\frac{5}{2} x^2$

Ví dụ 2. Xác định các số hữu tỉ a và $\mathrm{b}$ để đa thức $\mathrm{x}^3+\mathrm{ax}+\mathrm{b}$ chia hết cho đa thức $x^2+x-2$.

Giải : Cách 1. Đặt tính chia :

Để chia hết thì đa thức dư phải bằng 0 với mọi giá trị của $x$, nên :

$\left\{\begin{array}{l}a+3=0 \\ b-2=0\end{array} \Leftrightarrow\left\{\begin{array}{l}a=-3 \\ b=2\end{array}\right.\right.$

Vậy với $\mathrm{a}=-3 ; \mathrm{b}=2$ thì $\mathrm{x}^3+\mathrm{ax}+\mathrm{b}$ chia hết cho $\mathrm{x}^2+\mathrm{x}-2$.

Cách 2. (Phương pháp hệ số bất định)

Đa thức bị chia có bậc ba, đa thức chia có bậc hai nên thương là một nhị thức bậc nhất, hạng tử bậc nhất là $\mathrm{x}^3: \mathrm{x}^2=\mathrm{x}$.

Gọi thương là $\mathrm{x}+\mathrm{c}$, ta có :

$x^3+a x+b=\left(x^2+x-2\right)(x+c)$

nên

$x^3+a x+b=x^3+(c+1) x^2+(c-2) x-2 c $

Hai đa thức trên bằng nhau nên :

$\left\{\begin{array}{l}\mathrm{c}+1=0 \\ \mathrm{c}-2=\mathrm{a} \\ -2 \mathrm{c}=\mathrm{b}\end{array} \Leftrightarrow\left\{\begin{array}{l}\mathrm{c}=-1 \\ \mathrm{a}=-3 \\ \mathrm{~b}=2\end{array}\right.\right.$

Vậy với $\mathrm{a}=-3 ; \mathrm{b}=2$ thì $\mathrm{x}^3+\mathrm{ax}+\mathrm{b}$ chia hết cho $\mathrm{x}^2+\mathrm{x}-2$, thương là $\mathrm{x}-1$.

Cách 3. (Phương pháp xét giá trị riêng)

Gọi thương khi chia $\mathrm{x}^3+\mathrm{ax}+\mathrm{b}$ cho $\mathrm{x}^2+\mathrm{x}-2$ là $\mathrm{Q}(\mathrm{x})$, ta có :

$x^3+a x+b=(x-1)(x+2) Q(x)$

Vì đẳng thức đúng với mọi $x$ nên lần lượt cho $\mathrm{x}=1, \mathrm{x}=-2$, ta được :

$\left\{\begin{array}{l}1+a+b=0 \\ -8-2 a+b=0\end{array} \Leftrightarrow\left\{\begin{array}{l}a+b=-1 \\ -2 a+b=8\end{array} \Leftrightarrow\left\{\begin{array}{l}a=-3 \\ b=2 .\end{array}\right.\right.\right.$

Với $a=-3 ; b=2$ thì $x^3+a x+b$ chia hết cho $x^2+x-2$.

BÀI TẬP

Chia đơn thức cho đơn thức

71. Thực hiện phép tính :

a) $8^{12}: 4^6$;

b) $27^6: 9^2$;

c) $\frac{9^{15} \cdot 25^3 \cdot 4^3}{3^{10} \cdot 50^6}$

72. Chứng minh rằng biểu thức sau không âm với mọi giá trị của biến :

$A=\left(-15 x^3 y^6\right):\left(-5 x y^2\right)$

73. Chứng minh rằng giá trị của biểu thức sau không phụ thuộc vào giá trị của biến $\mathrm{y}(\mathrm{x} \neq 0 ; \mathrm{y} \neq 0)$ :

$B=\frac{2}{3} x^2 y^3:\left(-\frac{1}{3} x y\right)+2 x(y-1)(y+1)$

74. Tìm số tự nhiên $\mathrm{n}$ để đơn thức $\mathrm{A}$ chia hết cho đơn thức $\mathrm{B}$ :

$A=4 x^{n+1} y^2 ; B=3 x^3 y^{n-1}$

Chia đa thức cho dơn thức

75. Thực hiện phép tính :

a) $\left(\frac{1}{2} a^2 x^4+\frac{4}{3} a x^3-\frac{2}{3} a x^2\right):\left(-\frac{2}{3} a x^2\right)$

b) $4\left(\frac{3}{4} x-1\right)+\left(12 x^2-3 x\right):(-3 x)-(2 x+1)$.

76. Thực hiện phép tính rồi tìm giá trị nhỏ nhất của biểu thức :

$A=\left(9 x y^2-6 x^2 y\right):(-3 x y)+\left(6 x^2 y+2 x^4\right):\left(2 x^2\right) $

77. Tìm số tự nhiên $\mathrm{n}$ để đa thức $\mathrm{A}$ chia hết cho đơn thức $\mathrm{B}$ :

$A=7 x^{n-1} y^5-5 x^3 y^4 ; \quad B=5 x^2 y^n$

Chia đa thức cho đa thức

78. Rút gọn biểu thức

$\left[\left(x^3+y^3\right)-2\left(x^2-y^2\right)+3(x+y)^2\right]:(x+y)$

79. Chia các đa thức :

a) $\left(3 x^4-2 x^3-2 x^2+4 x-8\right):\left(x^2-2\right)$;

b) $\left(2 x^3-26 x-24\right):\left(x^2+4 x+3\right)$;

c) $\left(x^3-7 x+6\right):(x+3)$.

80. Xác định hằng số a sao cho :

a) $4 x^2-6 x+$ a chia hết cho $x-3$;

b) $2 \mathrm{x}^2+\mathrm{x}+\mathrm{a}$ chia hết cho $\mathrm{x}+3$;

c) $x^3+a x^2-4$ chia hết cho $x^2+4 x+4$.

81. Xác địṇh hằng số a sao cho :

a) $10 x^2-7 x+a$ chia hết cho $2 x-3$;

b) $2 x^2+a x+1$ chia cho $x-3$ dư 4 ;

c) $a x^5+5 x^4-9$ chia hết cho $x-1$.

82. Xác định các hằng số a và $\mathrm{b}$ sao cho :

a) $\mathrm{x}^4+\mathrm{ax}+\mathrm{b}$ chia hết cho $\mathrm{x}^2-4$;

b) $x^4+a x^3+b x-1$ chia hết cho $x^2-1$;

c) $x^3+a x+b$ chia hết cho $x^2+2 x-2$.

83. Xác định các hằng số a và b sao cho :

a) $x^4+a x^2+b$ chia hết cho $x^2-x+1$;

b) $a x^3+b x^2+5 x-50$ chia hết cho $x^2+3 x-10$;

c) $a x^4+b x^3+1$ chia hết cho $(x-1)^2$;

d) $x^4+4$ chia hết cho $x^2+a x+b$.

84. Tìm các hằng số $a$ và $b$ sao cho $x^3+a x+b$ chia cho $x+1$ thì dư 7 , chia cho $x-3$ thì dư $-5$.

85. Tìm các hằng số $\mathrm{a}, \mathrm{b}, \mathrm{c}$ sao cho $\mathrm{ax}^3+\mathrm{bx}^2+\mathrm{c}$ chia hết cho $\mathrm{x}+2$, chia cho $x^2-1$ thì dư $x+5$.

 

 

 

Một số vấn đề về nghiệm của đa thức

Bài viết của thầy Vương Trung Dũng

(Giáo viên chuyên toán trường Phổ thông Năng khiếu)

Trong những kì thi học sinh giỏi các bài toán về đa thức thường xuyên xuất hiện. Tuy nhiên trong chương trình THCS các kiến thức về đa thức chủ yếu dừng lại ở các khái niệm và các phép toán. Do đó khi vừa mới lên lớp 10 các kĩ năng của các em học sinh còn chưa cao. Bài viết này nhằm trình bày một vấn đề nhỏ về nghiệm của đa thức mà nội dung chính là Định lý Bézout và Định lý Viète, đối tượng hướng đến là các em học sinh cuối năm lớp 9 và đầu năm lớp 10.

Trong bài viết này ta kí hiệu $\mathbb{R}[x]$ là tập tất cả các đa thức có hệ số thực.

Cơ sở lý thuyết

Định lý Bézout. Cho $f(x) \in \mathbb{R}[x]$ và $a \in \mathbb{R}$. Số dư khi chia đa thức $f(x)$ cho đa thức $x-a$ là $f(a)$.

Theo thuật toán chia Euclide, tồn tại đa thức $g(x) \in \mathbb{R}[x]$ và số thực $r$ sao cho $$f(x)=(x-a)g(x)+r.$$
Trong đẳng thức trên thay $x=a$ vào hai vế ta được $f(a)=r.$ Từ đó ta có điều phải chứng minh.

Hệ quả 1. Đa thức $f(x)$ có nghiệm $x=a$ khi và chỉ khi $f(x)$ chia hết cho $x-a.$

Hệ quả 2. Nếu $a_1,a_2,…,a_n$ là các nghiệm của $f(x)$ thì $(x-a_1)(x-a_2)…(x-a_n)|f(x)$. Đặc biệt nếu $\deg f=n$ thì $f(x)=c(x-a_1)(x-a_2)…(x-a_n), c\in \mathbb{R}$.

Định lý 2. Một đa thức bậc $n$ có nhiều nhất là $n$ nghiệm. Đặc biệt nếu $\deg f \le n$ có quá $n$ nghiệm thì $f(x) =0.$
Hệ quả 3. Nếu $\deg f<n, \deg g<n$ mà tồn tại $n$ giá trị phân biệt của biến $x$ sao cho $f(x)=g(x)$ thì $f(x)= g(x) .$

Các ví dụ áp dụng.

Ví dụ 1. Biết đa thức $P(x)=x^5+x^2+1$ có 5 nghiệm phân biệt $x_1,x_2,x_3,x_4,x_5$. Đặt $Q(x)=x^2-2$. Tính $Q(x_1)Q(x_2)Q(x_3)Q(x_4)Q(x_5)$.

Lời giải

$P(x)$ có dạng $P(x)=(x-x_1)(x-x_2)(x-x_3)(x-x_4)(x-x_5)$. \

Ta có $$ \prod_{i=1}^{5} Q(x_i)=\prod_{i=1}^{5} (x_i^2-2)=\prod_{i=1}^{5} (\sqrt{2}-x_i) \prod_{i=1}^{5} (-\sqrt{2}-x_i)=P(\sqrt{2})P(-\sqrt{2})=-23. $$

Ví dụ 2. Cho $P(x) \in \mathbb{Z}[x]$ sao cho $|P(a)|=|P(b)|=|P(c)|=1$, với $a,b,c$ là các số nguyên đôi một khác nhau. Chứng minh đa thức $P(x)$ không có nghiệm nguyên.

Lời giải

Giả sử $P(x)$ có nghiệm nguyên $x_0$. Theo định lý Bézout $$ P(x)=(x-x_0)Q(x), \ \ \ \ (1) $$ với $Q(x) \in \mathbb{Z}[x]$. Từ đó suy ra $$ 1=|P(a)|=|a-x_0||Q(a)|. \ \ \ \ (2) $$
Do đó $|a-x_0|=1$, lập luận tương tự ta được $|b-x_0|=|c-x_0|=1$. Như vậy $a-x_0, b-x_0, c-x_0 \in \{-1,1\}$. Theo nguyên lý Dirichlet tồn tại hai trong ba số này bằng nhau từ đó tồn tại hai trong ba số $a,b,c$ bằng nhau, mâu thuẫn. Vậy $P(x)$ không có nghiệm nguyên.

Định lý Viete thuận. Cho đa thức $f \in \mathbb{R}[x]$, trong đó $$f(x)=a_nx^n+a_{n-1}x^{n-1}+…+a_1x+a_0,$$
trong đó $a_i \in \mathbb{R}$ và $a_n \ne 0.$ Giả sử rằng $x_1, x_2,…,x_n$ là các nghiệm (không nhất thiết phân biệt) của $f(x)$. Khi đó ta có

$x_1+x_2+…+x_n=-\dfrac{a_{n-1}}{a_n}$
$x_1x_2+x_1x_3+…+x_{n-1}x_n=\dfrac{a_{n-2}}{a_n}$

$x_1x_2…x_n=(-1)^n \dfrac{a_0}{a_n}$

Chứng minh

Định lý Viète có một ứng dụng rất lớn trong các bài toán về nghiệm của đa thức nhưng chứng minh của nó thì không hề khó. Thật vậy, vì $x_1, x_2,…,x_n$ là các nghiệm của $f$ nên ta có thể viết lại đa thức này dưới dạng $$f(x)=a_n(x-x_1)(x-x_2)…(x-x_n).$$
Khai triển vế phải rồi nhóm về dạng chuẩn tắc, sau đó so sánh hệ số của các số mũ tương ứng ở hai vế ta được điều phải chứng minh.

Lưu ý là định lý Viète vẫn đúng trong trường hợp $f$ không đủ $n$ nghiệm thực, nhưng do đối tượng của bạn đọc nên nội dung bài viết không đề cập đến.

Ví dụ 3. Tìm tất cả các giá trị của $a$ để nghiệm $x_1,x_2,x_3$ của đa thức $x^3-6x^2+ax+a$ thỏa mãn $$(x_1-3)^3+(x_2-3)^3+(x_3-3)^3=0.$$

Lời giải

Đặt $y=x-3$, khi đó $y_1=x_1-3, y_2=x_2-3, y_3=x_3-3$ là nghiệm của đa thức $$ (y+3)^3-6(y+3)^2+a(y+3)+a=y^3+3y^2+(a-9)y+4a-27. $$

Theo định lý Viète $$ \sum_{i=1}^{3} y_i=-3, \sum_{1 \le i<j \le 3} y_iy_j=-9, \prod_{i=1}^{3} y_i=27-4a. $$

Mặt khác theo giả thiết $\sum_{i=1}^{3} y_i^3=0$. Mà $$ \sum_{i=1}^{3} y_i^3=\Big(\sum_{i=1}^{3} y_i\Big)^3-3 \Big(\sum_{1 \le i<j \le 3} y_iy_j\Big)\Big(\sum_{i=1}^{3} y_i \Big)+3 \prod_{i=1}^{3} y_i. $$
Dô đó điều kiện cần và đủ của $a$ là $$ 0=(-3)^3-3(a-9)(-3)+3(27-4a)=-27-3a \Leftrightarrow a=-9. $$

Ví dụ 4. Chứng minh đa thức $P(x)=x^n+2nx^{n-1}+2n^2x^{n-2}+…+2n^{n-1}x+2n$ không thể có đủ $n$ nghiệm thực.

Lời giải
Giả sử $P(x)$ có đủ $n$ nghiệm thực $x_1,x_2,…,x_n$. Theo định lý Viet $$ \sum_{i}x_i=-2n, \sum_{i<j}x_ix_j=2n^2. $$
Khi đó $$ \sum_{i<j}x_ix_j=\dfrac{1}{2}(\sum_{i}x_i)^2-\dfrac{1}{2}\sum_ix_i^2 \le \dfrac{n-1}{2n}(\sum_{i}x_{i})^2=2n(n-1) <2n^2,$$
vô lí. Vậy ta có điều phải chứng minh.

Ta ký hiệu $$\begin{aligned}
\sigma_1 & = \sum_{i=1}^nx_i=-\dfrac{a_{n-1}}{a_n}, \sigma_2=\sum_{1 \le i < j \le n}^nx_ix_j =\dfrac{a_{n-2}}{a_n},…, \
\sigma_k & =\sum_{1 \le i_1 <i_2<…<i_k \le n}x_{i_1}x_{i_2}…x_{i_k}=(-1)^k \dfrac{a_{n-k}}{a_n}
\end{aligned}$$
và gọi $\sigma_k$ là các đa thức đối xứng bậc $k$ của các số $x_1,x_2,…,x_n$.

Định lý Viete đảo. Cho $x_1,x_2,…,x_n \in \mathbb{R}$. Gọi $\sigma_k$ là các đa thức đối xứng bậc $k$ của $n$ số đã cho. Khi đó $x_1,x_2,…,x_n$ là nghiệm của phương trình $$ X^n-\sigma_1X^{n-1}+\sigma_2X^{n-2}+…+(-1)^{n-1}\sigma_{1}X+(-1)^n \sigma_n=0.$$

Ví dụ 5. Gọi $a<b<c$ là 3 nghiệm của phương trình
$$x^3-3x+1=0.$$

a) Tính $A=\dfrac{1-a}{1+a}+\dfrac{1-b}{1+b}+\dfrac{1-c}{1+c};$
b) Tìm một đa thức bậc 3 nhận $a^2-2, b^2-2, c^2-2$ làm nghiệm;

Lời giải
a) Ta có
$$A+3=\dfrac{1-a}{1+a}+1+\dfrac{1-b}{1+b}+1+\dfrac{1-c}{1+c}+1=2\Big(\dfrac{1}{1+a}+\dfrac{1}{1+b}+\dfrac{1}{1+c}\Big).$$
Mặt khác, đặt $x=\dfrac{1}{1+a}$, khi đó $a=\dfrac{1}{x}-1.$ Vì $a^3-3a+1=0$ nên $$\Big(\dfrac{1}{x}-1\Big)^3-3\Big(\dfrac{1}{x}-1\Big)+1=0 \Leftrightarrow 3x^3-3x+1=0.$$
Từ đó suy ra $\dfrac{1}{1+a}, \dfrac{1}{1+b}, \dfrac{1}{1+c}$ là 3 nghiệm của phương trình trên, do đó $$\dfrac{1}{1+a}+\dfrac{1}{1+b}+\dfrac{1}{1+c}=0.$$ Vậy $A=-3.$
b) Theo định lý Viète $a+b+c=0, ab+bc+ca=-3$ và $abc=-1.$ Đặt $P(x)=x^3-3x+1=(x-a)(x-b)(x-c),$
ta có
\begin{eqnarray*}
a^2-2+b^2-2+c^2-2=a^2+b^2+c^2-6=(a+b+c)^2-2(ab+ac+bc)-6=0.
\end{eqnarray*}
Lại có
\begin{eqnarray*}
&&(a^2-2)(b^2-2)+(b^2-2)(c^2-2)+(c^2-2)(a^2-2)\\&=& a^2b^2+b^2c^2+c^2a^2-4(a^2+b^2+c^2)+12\\&=& (ab+bc+ca)^2-2abc(a+b+c) -3.6+12\\ &=&-3.
\end{eqnarray*}
Cuối cùng
\begin{eqnarray*}
&&(a^2-2)(b^2-2)(c^2-2)\\
&=& (\sqrt{2}-a)(\sqrt{2}+a)(\sqrt{2}+c)(-\sqrt{2}-a)(-\sqrt{2}-b)(-\sqrt{2}-c) \\
&=&P(\sqrt{2})P(-\sqrt{2})\\&=&-1.
\end{eqnarray*}
Theo định lý Viète đảo ta có $a^2-2, b^2-2, c^2-2$ là nghiệm của đa thức $x^3-3x+1=0.$

Bài tập có lời giải
Bài 1.  Định $m$ sao cho $F=x^3+y^3+z^3+mxyz$ chia hết cho $x+y+z$.

Lời giải
Xem F là một đa thức theo biến $x.$ Theo giả thiết $F(x) \vdots [x-(-y-z)]$ suy ra $$F(-y-z)=0 \Leftrightarrow (-y-z)^3+y^3+z^3+m(-y-z)yz \Leftrightarrow -yz(y+z)(3+m)=0, $$ với mọi $y,z \in \mathbb{R}$. Từ đó $m=-3.$

 

Bài 2.  (Canada 2001) Cho $P(x)$ là tam thức bậc hai có các hệ số nguyên thỏa mãn đồng thời:
i) Cả hai nghiệm đều nguyên;
ii) Tổng các hệ số là một số nguyên tố;
iii) Tồn tại số nguyên $k$ sao cho $P(k)=55$.

Chứng minh rằng $P(x)$ có một nghiệm bằng 2 và hãy tìm nghiệm còn lại.

Lời giải

Gọi $r_1 \le r_2$ là hai nghiệm. Ta có $P(x)=ax^2+bx+c=a(x-r_1)(x-r_2)$, từ đó $P(1)=a+b+c=a(1-r_1)(1-r_2)=p$ nên $a \in \{\pm 1, \pm p\}$.\

Nếu $a=p$ thì $(1-r_1)(1-r_2)=1$ nên $r_1=r_2=0$ hoặc $r_1=r_2=2$ (mâu thuẫn với (c) ).\

Nếu $a=-p$ thì $(1-r_1)(1-r_2)=-1$ nên $r_1=0, r_2=2$ (cũng mâu thuẫn).\

Vì $P(k)=a(k-r_1)(k-r_2)=-5.11$ nên ta được

$$\begin{cases}
a=1&\\
k-r_1=55&\\
k-r_2=1&
\end{cases} hay \ \begin{cases}
a=1&\\
k-r_1=11&\\
k-r_2=5&
\end{cases}$$

Trong trường hợp đầu tiên ta được $r_2=r_1+54, b=-2r_1-54$ và $c=r_1(r_1+54)$ do đó $r_1^2+52r_1-(53+p)=0$ nên $$ r_1=\frac{-52 \pm \sqrt{52^2+4(53+p)}}{2}= -26 \pm \sqrt{26^2+53+p }=-26 \pm \sqrt{ 27^2+p}.$$

Đặt $h^2=27^2+p \Leftrightarrow p=(h+27)(h-27)$, vì $p$ là nguyên tố nên $h-27=1 \Rightarrow h=28$ nhưng khi dó $p=55$ không là số nguyên tố.\

Trong trường hợp thứ hai $r_2=r_1+6$ nên $b=-2r_1-6$ và $c=r_1(r_1+6)$, do đó $p=10(2r_1+6)+r_1^2+6r_1$ hoặc $$ r_1^2+4r_1-(5+p)=0 \Leftrightarrow r=-1\pm \sqrt{3^2+p}. $$

Đặt $i^2=3^2+p \Leftrightarrow p=(i+3)(i-3), $ vì $p$ là số nguyên tố nên $i=4$ và do đó $p=7 \Rightarrow r_1=2, r_2=8.$

 

Bài 3.  Cho $P(x)=x^n+a_{n-1}x^{n-1}+…+a_1x+a_0$, trong đó $a_k =\pm 1$. Biết $P(x)$ có $n$ nghiệm, chứng minh rằng $n \le 3$.

Lời giải

Giả sử $x_1,…,x_n$ là các nghiệm của $P(x)$. Ta có $\displaystyle \sum_{i=1}^{n} x_i^2=3$ và $\dfrac{1}{x_1}, …, \dfrac{1}{x_n}$ là nghiệm của đa thức $Q(x)=a_0x^n+…+a_{n-1}x+1.$ Ta có $\displaystyle \sum_{i=1}^{n} \dfrac{1}{x_i^2}=3$. Suy ra $$ 9=\Big(\sum_{i=1}^{n} x_i^2 \Big)\Big(\sum_{i=1}^{n} \dfrac{1}{x_i^2}\Big) \ge n^2. $$
Do đó $n \le 3.$

Bài 4.  Cho các số thực $a,b,c$ và phương trình $x^4+4x^3+ax^2+bx+c=0$ có 4 nghiệm thỏa mãn $x_1^{16}+x_2^{16}+x_3^{16}+x_4^{16}=4$. Tìm các nghiệm đó.

Lời giải
Theo định lý Viète ta có $x_1+x_2+x_3+x_4=-4$.\
Áp dụng liên tiếp bất đẳng thức Cauchy Schwarz, ta được
\begin{eqnarray*}16&=&(x_1+x_2+x_3+x_4)^2\\ &\le& 4(x_1^2+x_2^2+x_3^2+x_4^2)\\ &\le& 4\sqrt{4(x_1^4+x_2^4+x_3^4+x_4^4)}\\ &\le& 4 \sqrt{4\sqrt{4(x_1^8+x_2^8+x_3^8+x_4^8)}}\\ &\le& 4 \sqrt{4 \sqrt{4\sqrt{4(x_1^{16}+x_2^{16}+x_3^{16}+x_{4}^{16})}}}=16. \end{eqnarray*}

Dấu "=" xảy ra khi và chỉ khi $x_1=x_2=x_3=x_4=-1$.

Bài 5. (VMO 1991)  Giả sử đa thức $P(x)=x^{10}-10x^9+39x^8+a_7x^7+...+a_1x+a_0$ với các hệ số thực $a_7, ..., a_0$ sao cho đa thức $P(x)$ có 10 nghiệm phân biệt. Chứng minh rằng các nghiệm này thuộc đoạn $[-\frac{5}{2},\frac{9}{2}].$

Lời giải

Gọi $x_1, x_2,…, x_{10}$ là các nghiệm của $P(x)$. Theo định lý Viète ta có
$$ \sum_{i=1}^{10} x_i=10 \ \text{và} \
\sum_{1 \le i <j \le 10} x_ix_j=39.$$

Do đó $$ \Big(\sum_{i=1}^{10} x_i \Big)^2=\sum_{i=1}^{10} x_i^2+2 \sum_{1 \le i<j \le 10} x_ix_j \Rightarrow \sum_{i=1}^{10} x_{i}^2=100-2.39=22. $$

Hơn nữa $$ \sum_{i=1}^{10} (x_i-1)^2=\sum_{i=1}^{10} x_i^2-2 \sum_{i=1}^{10} x_i+10=12 \Rightarrow (x_i-1)^2 \le 12 <(3.5)^2 ,$$
với mọi $i=1,2,…,10.$
Từ đó suy ra điều phải chứng minh.

Bài 6.  Cho các số thực $a,b$ trong đó $a \ne 0.$ Chứng minh rằng tất cả các nghiệm của phương trình $$ax^4+bx^3+x^2+x+1=0$$ không đồng thời là nghiệm thực.

Lời giải
Gọi $\alpha_1, \alpha_2, \alpha_3, \alpha_4$ lần lượt là các nghiệm của phương trình đã cho. Dễ thấy các nghiệm này đều khác 0 và có tích bằng $\dfrac{1}{a}.$ Khi đó nghiệm của phương trình $x^4+x^3+x^2+bx+a=0$ lần lượt là $$\beta_1=\dfrac{1}{\alpha_1}, \beta_2=\dfrac{1}{\alpha_2},\beta_3=\dfrac{1}{\alpha_3},\beta_4=\dfrac{1}{\alpha_4}.$$
Theo định lí Viète $$\sum_{j=1}^{4} \beta_j=-1, \sum_{1 \le j<k \le 4}\beta_j \beta_k=1.$$
Dẫn đến
$$\sum_{j=1}^{4}\beta_j^2=\Big(\sum_{j=1}^{4}\beta_j\Big)^2-2 \Big(\sum_{1 \le j<k \le 4}\beta_j \beta_k\Big)=1-2=-1.$$
Vô lí, bài toán được chứng minh xong.

Bài 7. Giả sử đa thức $ax^3-x^2+bx-1=0$ có 3 nghiệm dương phân biệt. Chứng minh rằng:

a) $0<3ab \le 1;$
b) $b \ge 9a;$
c) $b \ge \sqrt{3}.$

Lời giải
a) Gọi $x_1, x_2, x_3$ là 3 nghiệm của đa thức đã cho. Khi đó theo Định lý Viète, ta có $$x_1+x_2+x_3=\dfrac{1}{a}, x_1x_2+x_1x_3+x_2x_3=\dfrac{b}{a}, x_1x_2x_3=\dfrac{1}{a}.$$
Từ đó suy ra $a>0$ nên $b>0$, dẫn đến $ab>0.$ Từ bất đẳng thức $$(x_1+x_2+x_3)^2 \ge 3(x_1x_2+x_1x_3+x_2x_3)$$ ta được $\dfrac{1}{a^2} \ge 3.\dfrac{b}{a}$ dẫn đến $0 <3ab \le 1.$
b) Vì $(x_1+x_2+x_2)(x_1x_2+x_1x_3+x_2x_3) \ge 9x_1x_2x_3$ nên $\dfrac{b}{a^2} \ge \dfrac{9}{a},$ dẫn đến $b \ge 9a.$
c) Theo bất đẳng thức $(x_1x_2+x_1x_3+x_2x_3)^2 \ge 3x_1x_2x_3(x_1+x_2x+x_3)$ ta được $\dfrac{b^2}{a^2} \ge \dfrac{3}{a^2}$. Dẫn đến $b^2 \ge 3$ và vì $b \ge 0$ nên $b \ge \sqrt{3}.$

Bài 8.  Cho đa thức $x^3+\sqrt{3}(a-1)x^2-6ax+b=0$ có 3 nghiệm thực. Chứng minh rằng $$|b| \le |a+1|^3.$$

Lời giải
Gọi $x_1, x_2, x_3$ là 3 nghiệm của đa thức đã cho, theo định lý Viète $$x_1+x_2+x_3=-\sqrt{3}(a-1), x_1x_2+x_2x_3+x_1x_3=-6a, x_1x_2x_3=-b.$$
Ta có
\begin{eqnarray*}
\sqrt[3]{|b|}= \sqrt[3]{|x_1|.|x_2||x_3|} &\le& \sqrt{\dfrac{x_1^2+x_2^2+x_3^2}{3}} \\&=& \sqrt{\dfrac{(x_1+x_2+x_3)^2-2(x_1x_2+x_2x_3+x_1x_3)}{3}}\\&=& \sqrt{\dfrac{3(1-a)^2+12a}{3}}\\&=& |a+1|.
\end{eqnarray*}
Suy ra $|b| \le |a+1|^3,$ điều phải chứng minh.

 

Bài 9.  [Mathematical Reflections S455] Cho $a,b \in \mathbb{R}$ sao cho tất cả các nghiệm của đa thức
$$P(x)=x^4-x^3+ax+b$$ có 4 nghiệm thực.
a)  Chứng minh rằng $a+ b \ge 0;$
b) Chứng minh rằng $P \Big(-\dfrac{1}{2}\Big) \le \dfrac{3}{16}.$

Lời giải
a) Gọi $x_1, x_2, x_3, x_4$ là 4 nghiệm của đa thức đã cho. Theo định lý Viète ta có
\begin{eqnarray*}
&&x_1+x_2+x_3+x_4=1 \\&& x_1x_2+x_1x_3+x_1x_4+x_2x_3+x_2x_4+x_3x_4=0\\&&-x_1x_2x_3x_4\Big(\dfrac{1}{x_1}+\dfrac{1}{x_2}+\dfrac{1}{x_3}+\dfrac{1}{x_4}\Big)=a\\&&x_1x_2x_3x_4=b.
\end{eqnarray*}
Từ hai phương trình đầu ta được $$x_1^2+x_2^2+x_3^2+x_4^2=1.$$
Theo bất đẳng thức Cauchy-Schwarz
$$1=x_1^2+(x_2^2+x_3^2+x_4^2) \ge x_1^2+\dfrac{1}{3}(x_2+x_3+x_4)^2=x_1^2+(1-x_1)^2.$$
Từ đó ta có $$-\dfrac{1}{2} \le x_1 \le 1.$$
Hoàn toàn tương tự $-\dfrac{1}{2}\le x_2, x_3, x_4 \le 1.$ Khi đó vì $P(x)=(x-1x_1)(x-x_2)(x-x_3)(x-x_4)$ nên dễ thấy $$P(1) \ge 0 \Leftrightarrow a+b \ge 0.$$
b) Bây giờ ta cần chứng minh $$P\Big(-\dfrac{1}{2}\Big) \le \dfrac{3}{16} \Leftrightarrow a \ge 2b.$$
Nếu $b \le 0$ thì từ $a+b \ge 0$ ta suy ra $a \ge 0$ nên hiển nhiên nhiên $a \ge 2b.$ Giả sử $b >0,$ thế thì $x_1x_2x_3x_4 >0$ và do đó ta có
$$a \ge 2b \Leftrightarrow \dfrac{1}{x_1}+\dfrac{1}{x_2}+\dfrac{1}{x_3}+\dfrac{1}{x_4} \le -2. \ \ \ \ (1)$$
Trong trường hợp này phải có hai nghiệm là số dương và hai nghiệm là số âm. Không mất tổng quát giả sử $x_1, x_2>0$ và $x_3, x_4<0$. Vì $-\dfrac{1}{2} \le x_4 \le 1$ nên $2x_4+1 \ge 0, 1-x_4 \ge 0$ và $x_1x_2x_3 <0$. Dẫn đến
\begin{eqnarray*}
x_4^2(1-x_4) \ge x_1x_2x_3(2x_4+1) &\Leftrightarrow& x_4^2(x_1+x_2+x_3) -x_1x_2x_3 \ge 2x_1x_2x_3x_4\\
&\Leftrightarrow& \dfrac{x_4(x_1+x_2+x_3)}{x_1x_2x_3} -\dfrac{1}{x_4} \ge 2\\ &\Leftrightarrow& \dfrac{1}{x_1}+\dfrac{1}{x_2}+\dfrac{1}{x_3}+\dfrac{1}{x_4} \le -2.
\end{eqnarray*}
Bất đẳng thức (1) được chứng minh xong.

Bài 10. Cho số tự nhiên $k>0$ và hai số thực $a, b$ sao cho $x^k + ax + 1$ chia hết cho $x^2 + bx + 1$ và phương trình $x^2 + bx + 1 = 0$ có hai nghiệm. Chứng minh $a(a-b)=0$.

Lời giải
Theo giả thiết tồn tại đa thức $P(x) \in \mathbb{R}[x]$ sao cho $ x^k + ax + 1 = P(x)(x^2 + bx + 1) \ (1).$ Gọi $r_1, r_2$ là hai nghiệm của phương trình $x^2 + bx + 1 = 0$. Khi đó $$(x – r_1)(x – r_2) = x^2 + bx + 1.$$

Theo định lý Viète $\begin{cases}
r_1 + r_2 = -b&\\
r_1r_2 = 1.&
\end{cases}$
Thay vào (1) ta được $$0 = \sum_{i=1}^2 \Big( r_i^k + ar_i + 1 \Big) = r_1^k + r_2^k + a(r_1 + r_2) + 2,$$
suy ra $$r_1^k + r_2^k = -a(r_1 + r_2) – 2 = ab – 2$$ và do đó $$ r_1^k + r_2^k = -a(r_1 + r_2) – 2 = ab – 2.$$
Sử dụng (1) một lần nữa ta được $$a^2r_1r_2 = (r_1^k + 1)(r_2^k + 1) = (r_1r_2)^k + r_1^k + r_2^k + 1.$$
Suy ra $a^2 . 1 = 1^k + (ab – 2) + 1 = ab \Leftrightarrow a(a-b)=0.$

Bài 11.  Cho $P(x) $ là một đa thức hệ số nguyên thỏa mãn các phương trình $P(x)=1, P(x)=2, P(x)=3$ có ít nhất một nghiệm nguyên lần lượt là $x_1, x_2, x_3$.

a) Chứng minh $x_1, x_2, x_3$ là nghiệm nguyên duy nhất của các phương trình trên.
b) Chứng minh rằng phương trình $P(x)=5$ có tối đa một nghiệm nguyên.

Lời giải

a) Vì phương trình $P(x)=2$ nhận $x=x_2$ làm nghiệm nên $$ P(x)=(x-x_2)q(x)+2 \ \ \ \ (1). $$

Vì $P(x)$ là đa thức với hệ số nguyên mà $x_2$ nguyên nên $q(x) \in \mathbb{Z}[x]$. Trong (1) lân lượt thay $x$ bởi $x_1, x_3$ ta được $$ \begin{cases}
1=P(x_1)=(x_1-x_2)q(x_1)+2&\\
3=P(x_3)=(x_3-x_2)q(x_3)+2.&
\end{cases} \Leftrightarrow \begin{cases}
(x_1-x_2)q(x_1)=-1&\\
(x_3-x_2)q(x_3)=1&
\end{cases}.$$
Hơn nữa $x_1 \ne x_3$ nên $\begin{cases}
x_1-x_2=1&\\
x_3-x_2=-1&
\end{cases}$ hoặc $\begin{cases}
x_1-x_2=-1&\\
x_3-x_2=1.&
\end{cases}$\

Trong hai trường hợp ta đều có $x_2=\dfrac{x_1+x_3}{2}$. Giả sử phương trình $P(x)=2$ còn có nghiệm nguyên $x_2′ \ne x_2$ áp dụng lại lập luận trên ta lại có $x_2’=\dfrac{x_1+x_3}{2}=x_2,$ mâu thuẫn. Vậy phương trình này chỉ có một nghiệm nguyên duy nhất là $x_2.$\

Tương tự cho hai phương trình còn lại.

b) Xét phương trình $P(x)=5$.\

Nếu phương trình này không có nghiệm nguyên thì bài toán là hiển nhiên.\

Nếu phương trình này có một nghiệm nguyên $x_5$ thì từ (1) suy ra $$ 5=P(x_5)=(x_5-x_2)q(x_5)+2 \Rightarrow (x_5-x_2)q(x_5)=3. $$

Suy ra $x_5-x_2 \in \{\pm 1, \pm 3\}$.\

Nếu $x_5-x_2=\pm 1$ thì $x_5$ phải trùng với $x_1$ hoặc $x_3$, vô lý.\

Nếu $x_5-x_2= \pm 3$. Vì phương trình $P(x)=3$ nhận $x_3$ làm nghiệm nên $$P(x)=(x-x_3)r(x)+3 \Rightarrow 5=P(x_5)=(x_5-x_3)r(x_5)+3.$$
Để ý rằng $r(x) \in \mathbb{Z}[x]$ nên từ $(x_5-x_3)r(x_5)=2$ nên $x_5-x_3 \in \{\pm 1, \pm 2\}$. Xét hai khả năng:

Trường hợp 1. $\begin{cases}
x_1-x_2=1&\\
x_3-x_2=-1&
\end{cases} \Leftrightarrow \begin{cases}
x_1=1+x_2&\\
x_3=-1+x_2&
\end{cases}$\

– Nếu $x_5-x_2=3 \Rightarrow x_5-x_3=3=(3+x_2)-(-1+x_2)=4$, mâu thuẫn.\

– Nếu $x_5-x_2=-3 \Rightarrow x_5-x_3=(-3+x_2)-(-1+x_2)=-2$, thỏa mãn.\

Tóm lại nếu $\begin{cases}
x_1-x_2=-1&\\
x_3-x_2=1&
\end{cases} \Rightarrow x_5-x_2=-3 \Rightarrow x_5=x_2-3$. Như thế $x_5$ xác định theo $x_1, x_2, x_3$ là duy nhất.\

Trường hợp 2.

Tương tự nếu $$\begin{cases}
x_1-x_2=-1&\\
x_3-x_2=1&
\end{cases} \Rightarrow x_5-x_2=3 \Rightarrow x_5=x_2+3. $$

Như vậy nghiệm nguyên của phương trình này nếu có là duy nhất, bài toán được chứng minh xong.

 

Bài tập rèn luyện

  1. Giả sử đa thức $P(x), Q(x), R(x), S(x) \in \mathbb{R}[x]$ thỏa mãn dẳng thức $$ P(x^5)+xQ(x^5)+x^2R(x^5)=(x^4+x^3+x^2+x+1)S(x).$$
    Chứng minh rằng $P(x)$ chia hết cho $x-1$.
    a) Biết tích của hai trong bốn nghiệm của phương trình $x^4-18x^3+kx^2+200x-2016=0$ là $-32.$ Tìm $k$ .
    b) Biết đa thức $$P(x)=x^n-2nx^{n-1}+2n(n-1)x^{n-2}+...+a_0$$ có $n$ nghiệm thực. Tìm tất cả các nghiệm này.
  2. Giả sử đa thức $P(x)=ax^n-ax^{n-1}+c_2x^{n-2}+...+c_{n-2}x^2-n^2bx+b$ có đúng $n$ nghiệm dương. Chứng minh rằng tất cả các nghiệm này bằng nhau.
    a) Giả sử $x_1, x_2$ là hai trong bốn nghiệm của đa thức $P(x)=x^4+x^3-1$. Chứng minh rằng $x_1x_2$ là nghiệm của đa thức $Q(x)=x^6+x^4+x^3-x^2-1$.
    b) Tìm tất cả các cặp số thực $a,b$ sao cho các đa thức $$P(x)=x^4+2ax^2+4bx+a^2 \ \text{và} \ Q(x)=x^3+ax+b$$ có chung hai nghiệm thực phân biệt.
  3. Cho đa thức $f(x)=3x^3-5x^2+2x-6$ có các nghiệm là $\alpha, \beta, \gamma$. Tính $$T=\Big(\dfrac{1}{\alpha-2}\Big)^2+\Big(\dfrac{1}{\beta-2}\Big)^2+\Big(\dfrac{1}{\gamma-2}\Big)^2.$$
  4. Gọi $r_1, r_2,...,r_7$ là các nghiệm phân biệt của đa thức $P(x)=x^7-7$. Đặt $\displaystyle K=\prod_{1 \le i<j \le 7}(r_i+r_j)$. Tính $K^2.$

 

Tài liệu tham khảo

  1. Phan Huy Khải, Đa thức.
  2. Nguyễn Hữu Điển, Đa thức và ứng dụng.
  3. Titu Andresscu, Navid Safaei, Alessandro Ventullo, Polynomial Problems.
  4. Tạp chí Mathematical Reflections.

 

Đa thức bất khả quy

ĐA THỨC BẤT KHẢ QUY

(Thầy Vương Trung Dũng  giáo viên trường PTNK TP Hồ Chí Minh)

1. Giới thiệu sơ lược 

Đa thức bất khả qui là một vấn đề kinh điển trong đa thức nói riêng và trong toán học nói chung. Các bài toán về đa thức bất khả qui cũng thường xuyên xuất hiện trong các kì thi Olympic về toán. Người ta quan tâm nhiều nhất về tính bất khả qui của một đa thức trên vành $\mathbb{Z}[x]$ và $\mathbb{Q}[x]$. Có nhiều cách để kiểm tra tính bất khả qui của một đa thức loại này chẳng hạn như dùng trực tiếp định nghĩa hoặc dùng các tiêu chuẩn như tiêu chuẩn Eisenstein, tiêu chuẩn Perron, tiêu chuẩn Cohn, tiêu chuẩn Dumas… tuy nhiên bài viết này chỉ đề cập đến hai phương pháp thường được sử dụng nhất là sử dụng trực tiếp định nghĩa và tiêu chuẩn Eisenstein và các dạng mở rộng của nó cùng với đó là một kĩ thuật tối quan trọng là rút gọn theo một modulo nguyên tố $p$. Các tiêu chuẩn khác hi vọng sẽ có dịp trình bày trong một bài viết khác.

Trong tài liệu này ta qui ước $\mathbb{Z}_p=\mathbb{Z}/p\mathbb{Z}$ và $\mathbb{K}$ là một trong các tập $\mathbb{Z},\ \mathbb{Q},\ \mathbb{R}, \ \mathbb{Z}_p$. Khi đó, $ \mathbb{K}[x]$ (tương ứng $ \mathbb{K}[x,y]$) là các vành đa thức một biến (tương ứng 2 biến) có hệ số trong $ \mathbb{K}$.

Định nghĩa 1.1:  Đa thức $P(x)$ trong vành $\mathbb{K}[x]$ được gọi là khả qui trên $\mathbb{K}$ nếu $P(x)=f(x).g(x)$ trong đó $f(x), g(x)$ là các đa thức không khả nghịch trong $\mathbb{K}[x]$. Đa thức $P(x)$ được gọi là bất khả qui nếu $P(x)$ không khả nghịch và không khả qui.

Nói riêng, khi $\mathbb{K}$ là một trường thì một đa thức $P(x) \in \mathbb{K}[x]$ có bậc dương được gọi là khả qui trên $\mathbb{K}$ nếu có thể phân tích được thành tích hai đa thức có bậc dương trong $\mathbb{K}[x]$, ngược lại $P(x)$ được gọi là bất khả qui trên $\mathbb{K}$.

Định lí Gauss 1.1: Các vành đa thức

  • $\mathbb{R}[x], \ \mathbb{C}[x],\ \mathbb{Q}[x], \ \mathbb{Z}_p[x]$
  •  $\mathbb{Z}[x], \ \mathbb{Z}[x,y], \ \mathbb{Q}[x,y]…$

là có sự phân tích duy nhất thành các nhân tử bất khả qui và sự phân tích này là duy nhất. Nói riêng các khái niệm về đa thức bất khả qui, ước chung lớn nhất, bội chung nhỏ nhất vẫn còn đúng trên các vành này.

Lưu ý: Trong trường hợp $1$ ở trên là các đa thức có hệ số trên trường nên trên đó thuật toán Euclid hay định lí Bezout vẫn còn đúng nhưng trường hợp $2$ thì không.

2. Tính bất khả qui trên $\mathbb{C}[x]$ và $\mathbb{R}[x]$

Định lí 2.1: Mọi đa thức có bậc lớn hơn 1 đều khả qui trên $\mathbb{C}[x]$.

Chứng minh

Giả sử $degP>1$. Ta thừa nhận định lí cơ bản của đại số “Mọi đa thức $P(x) \in \mathbb{C}[x]$ có bậc lớn hơn 1 đều có ít nhất một nghiệm trên $\mathbb{C}$”. Khi đó $P(x)$ có nghiệm $x_0 \in \mathbb{C}$ nên theo Định lí Bezout $$P(x)=(x-x_0)Q(x),$$

trong đó $deg\ge 1$ nên $P(x)$ khả qui trên $\mathbb{C}[x].$

Định lí 2.2: Mọi đa thức có hệ số thực bậc lớn hơn 2 đều khả qui trên $\mathbb{R}[x]$. Nói riêng một đa thức là bất khả qui trên $\mathbb{R}[x]$ khi và chỉ khi nó là đa thức bậc nhất hoặc bậc 2 vô nghiệm.

Chứng minh

Giả sử $P \in \mathbb{R}[x]$ và $deg P >2$.

  • Nếu $\deg P$ lẻ thì $P$ có ít nhất một nghiệm thực nên nó khả qui.
  • Nếu $\deg P$ chẵn thì $P$ có một nghiệm phức $\alpha$, khi đó $\overline{\alpha}$ cũng là nghiệm của $P$ và do đó $P(x)=(x-\alpha)(x-\overline{\alpha})Q(x)$ là khả qui.

3. Tính bất khả qui trên $\mathbb{Z}[x]$ và $ \mathbb{Q}[x]$

Qua Định lí 2.1 và Định lí 2.2 ta thấy nếu $\mathbb{K}=\mathbb{C},\ \mathbb{R}$ thì tính bất khả quy là đơn giản nên ta quan tâm đến trường hợp $\mathbb{K}=\mathbb{Z}, \ \mathbb{Q}.$ Thật may mắn là bổ đề Gauss mà ta trình bày sau đây sẽ cho ta một sự tương ứng về tính bất khả qui của một đa thức hệ số nguyên trên $\mathbb{Z}[x]$ và $\mathbb{Q}[x]$.

Định nghĩa 3.1: Cho đa thức $P(x)=a_nx^n+a_{n-1}x^{n-1}+…+a_1x+a_0 \in \mathbb{Z}[x]$, đa thức $P$ được gọi là nguyên bản nếu $gcd(a_n,…,a_0)=1$

Mệnh đề 3.1: Tích của hai đa thức nguyên bản là một đa thức nguyên bản.

Mệnh đề 3.2: Mọi đa thức $P \in \mathbb{Q}[x]$ đều viết được dưới dạng $P=cP_0(x)$, trong đó $P_0$ là một đa thức nguyên bản và $c_0 \in \mathbb{Q}.$

Định lí 3.1 (Bổ đề Gauss):  Một đa thức hệ số nguyên, có bậc dương bất khả qui trong $\mathbb{Q}[x]$ khi và chỉ khi nó bất khả qui trong $\mathbb{Z}[x]$.

Chứng minh

Hiển nhiên nếu $P(x)$ bất khả qui trên $\mathbb{Q}[x]$ sẽ bất khả qui trên $\mathbb{Z}[x]$.

Ngược lại giả sử $P(x)$ bất khả qui trên $\mathbb{Z}[x]$ mà

$P(x)=P_1(x)P_2(x)$, với $P_1, P_2 \in \mathbb{Q}[x]$ và $1\le deg \ P_1, degP_2 \le deg\ P$.

Khi đó ta viết lại $P_1=\dfrac{a_1}{b_1}Q_1(x), P_2=\dfrac{a_2}{b_2}Q_2(x)$, với $(a_i,b_i)=1$ và $Q_i$ nguyên bản, $i \in \{1,2\}$.

Suy ra $P(x)=\dfrac{a_1a_2}{b_1b_2}Q_1(x)Q_2(x)=\dfrac{p}{q}Q_1(x)Q_2(x),$ trong đó $p=a_1b_1, q=a_2b_2$ và $ (p,q)=1.$

Do $P\in \mathbb{Z}[x]$ nên các hệ số của $Q_1(x)Q_2(x)$ phải chia hết cho $q$ điều này trái với tính nguyên bản của $Q_1(x)Q_2(x)$.

Từ đó ta có điều phải chứng minh.

Định lí 3.2: Cho $P(x)=a_nx^n+a_{n-1}x^{n-1}+…+a_1x+a_0 \in \mathbb{Z}[x]$. Giả sử $P$ có nghiệm hữu tỉ $x=\dfrac{p}{q}$ với $(p,q)=1$. Khi đó, $p$ là ước của $a_0$ còn $q$ là ước của $a_n.$ Nói riêng, mọi nghiệm hữu tỉ của một đa thức monic (đơn khởi, hệ số của bậc cao nhất bằng $\pm1$) với hệ số nguyên đều là nghiệm nguyên.

Chứng minh

Giả sử $P(x)$ có nghiệm hữu tỉ $\dfrac{p}{q},$ với $ (p,q)=1$. Khi đó $$a_n(\dfrac{p}{q})^n+…+a_1. \dfrac{p}{q}+a_0=0,$$

qui đồng mẫu số ta được $$a_np^n+…+a_0q^n=0.$$

Vì vế phải chia hết cho $p$ nên vế trái chia hết cho $p$, từ đó suy ra $a_0q^n$ chia hết cho $p$, lại có $(q^n,p)=1$ nên $a_0$ chia hết cho $p$. Lập luận tương tự ta được $a_n$ chia hết cho $q$.

Đinh lí 3.3: Cho $P \in \mathbb{Q}[x]$ có bậc 2 hoặc 3. Khi đó, $P(x)$ là bất khả qui khi và khi khi $P(x)$ không có nghiệm hữu tỉ.

Chứng minh

Hiển nhiên nếu $P$ có nghiệm hữu tỉ thì nó khả qui.

Đảo lại, nếu $P$ khả qui thì $P$ phân tích được thành tích của hai đa thức hữu tỉ.

Điều kiện bậc của $P$ bằng 2 hoặc 3 chứng tỏ một trong hai nhân tử trên phải có bậc 1.

Từ đó suy ra $f$ có nghiệm hữu tỉ.

Lưu ý: Định lí trên vẫn còn đúng nếu ta thay $\mathbb{Q}$ bởi một trường $\mathbb{K}$ bất kì. Tức là, đa thức $f \in \mathbb{K}[x]$ với bậc bằng 2 hoặc 3 là bất khả qui nếu và chỉ nếu nó không có nghiệm trong $\mathbb{K}.$

Dưới đây là một số ví dụ

Ví dụ 3.1 (Định lí Schur): Cho các số nguyên phân biệt $a_1, a_2,…,a_n$. Khi đó đa thức

a) $f(x)=(x-a_1)(x-a_2)…(x-a_n)-1$ là bất khả qui trên $\mathbb{Q}[x]$.

b) $f(x)=(x-a_1)(x-a_2)…(x-a_n)+1$ là bất khả qui trên $\mathbb{Q}[x]$ ngoại trừ các trường hợp

  • $(x-a)(x-a-2)+1=(x-a-1)^2$,
  • $(x-a)(x-a-1)(x-a-2)(x-a-3)+1=[(x-a-1)(x-a-2)+1]^2.$
Giải

a) Giả sử $(x-a_1)(x-a_2)…(x-a_n)-1=g(x)h(x)$, với $1 \le deg \ f, \deg \ g \le n-1$ và $g, h \in \mathbb{Z}[x].$

Ta có $g(a_i)h(a_i)=-1$ với mọi $i$, từ đó do $g(a_i), h(a_i)$ là các số nguyên nên $ (g+h)(a_i)=0,$ với mọi $i=1,2,…,n.$

Nhưng vì $deg \ (g+h) \le n-1$ triệt tiêu tại $n$ giá trị phân biệt nên $g \equiv – h$.

Từ đó $$(x-a_1)(x-a_2)…(x-a_n)-1=-(g(x))^2.$$

Đẳng thức trên là vô lí vì hệ số cao nhất ở hai vế trái dấu.

b) Lập luận hoàn toàn như trên, giả sử $f(x)$ là khả qui, bằng một phép đổi biến đơn giản ta hoàn toàn có thể viết lại $f$ dưới dạng $$f(x)=x(x-a_1)(x-a_2)…(x-a_{n-1})+1=g(x).h(x),$$ trong đó $0<a_1<a_2<…<a_{n-1}$ và $1 \le g, h \in \mathbb{Z}, \deg(g), \deg(h) \le n-1$.

Từ đẳng thức $g(a_i)h(a_i)=1$ ta suy ra $g(a_i)=h(a_i)= \pm 1$ với mọi $i$ và đẳng thức này xảy ra tại $n$ giá trị phân biệt. Điều đó dẫn dến $g(x)=h(x)$ và ta có $f(x)=g^2(x)$.

Nói cách khác, $\deg(f)=n$ là một số chẵn. Khi đó $f(\dfrac{1}{2})=\dfrac{1}{2}(\dfrac{1}{2}-a_1)…(\dfrac{1}{2}-a_{n-1})+1=1-\dfrac{1}{2^n}(2a_1-1) \ldots (2a_{n-1}-1) \le 1-\dfrac{1}{2^n}1.3 \ldots (2n-3) <0$ với mọi $n \ge 6$ (vô lí). Như vậy $n=2$ hoặc $n=4$.

  • Nếu $n=2$ thì $f(\dfrac{1}{2})=1-\dfrac{1}{4}(2a_1-1) \Rightarrow a_1 \le \dfrac{5}{2}$ và do đó $a_1=1, 2$. Giá trị $a_1$ cho ta $f(x)=x(x-1)+1$ là bất khả qui và $a_1=2$ cho ta $f(x)=x(x-2)+1=(x-1)^2$ là khả qui.
  • Nếu $n=4$ thì $f(\dfrac{1}{2})=1-\dfrac{1}{16}(2a_1-1)(2a_2-1)(2a_3-1) \Rightarrow 0 \le \dfrac{1}{16}(2-1)(3-1)(2a_3-1) \Rightarrow a_3 \le \dfrac{19}{6}$. Xét trường hợp $a_1=1, a_2=2, a_3=3$ ta được $$f(x)=x(x-1)(x-2)(x-3)+1=(x^2-3x+1)^2$$ là khả qui.

Bài toán được chứng minh xong.

Ví dụ 3.2: Cho $a_1, a_2,…,a_n$ là các số nguyên dương phân biệt. Chứng minh rằng đa thức $$P(x)=(x-a_1)^2(x-a_2)^2…(x-a_n)^2+1$$ là bất khả qui trên $\mathbb{Z}.$

Giải

Giả sử $P(x)$ là khả qui, tức tồn tại hai đa thức $G(x), H(x) \in \mathbb{Z}[x]$ có bậc không bé hơn 1 sao cho $P(x)=G(x).H(x)$.

Ta có $P(a_i)=G(a_i).H(a_i)$ với $i=1,2,…,n$ nên $G(a_i)=H(a_i)= \pm 1.$ Ta xét các trường hợp

  • Nếu $deg \ G= \ deg \ H$ thì $deg(G-H) \le n-1 \Rightarrow G \equiv H.$ Từ đó $P(x)=(G(x))^2 \Leftrightarrow 1= \Big(G(x)-(x-a_1)…(x-a_n) \Big)\Big(G(x)+(x-a_1)…(x-a_n) \Big)$ , vô lí vì bậc vế phải luôn không nhỏ thua 1.
  • Nếu $degH<deg G$ thì $degH<n$ mà $H(a_i)= \pm1, i=1,2,…,n$ dẫn đến $H$ là đa thức hằng, vô lí.

Vậy $P(x)$ bất khả qui.

4. Rút gọn modulo $p$ nguyên tố

Kĩ thuật rút gọn modulo $p$ nguyên tố là một kĩ thuật tối quan trọng trong việc chứng minh một đa thức là bất khả qui trên $\mathbb{Z}$. Nó đưa các hệ số từ một trường vô hạn các phần tử về một trường hữu hạn các phần từ, từ đó các tính toán của ta có thể được đơn giản hơn.

Định nghĩa 4.1: Cho $P(x)= \sum \limits_{i=0}^n a_ix_i \in \mathbb{Z}[x], a_n \ne 0$ và $p$ là số nguyên tố. Giả sử $p$ không phải là ước của $gcd(a_1,a_2,…,a_n)$. Ta kí hiệu $\overline{P}$ là đa thức nhận được từ $P$ bằng cách rút gọn các hệ số theo modulo $p$ (lúc này $P(x) \in \mathbb{Z}_p[x]$). Khi đó ta gọi $\overline{P}$ là \textit{đa thức rút gọn theo modulo} $p$ của $P.$

Từ định nghĩa trên ta có sự kiện sau là hiển nhiên $$\overline{P+Q}=\overline{P}+\overline{Q}$$

$$\overline{PQ}=\overline{P}. \ \overline{Q}$$

Định nghĩa 4.2: Nếu đa thức rút gọn modulo $p$ của P bất khả qui thì ta nói đa thức $P$ bất khả qui $mod \ p.$

Định lí 4.1: Với mỗi $P(x) \in \mathbb{Z}[x]$, tồn tại các đa thức $P_1(x), P_2(x), …, P_k(x) \in \mathbb{Z}_p[x]$ sao cho $$\overline{P}(x)=P_1(x).P_2(x)…P_k(x),$$

sự phân tích này là duy nhất theo modulo $p$.

Định lí 4.2: Cho $P(x)= \sum \limits_{i=0}^n a_ix_i \in \mathbb{Z}[x], a_n \ne 0$ và $p$ không là ước của $a_n$. Khi đó, nếu $P(x)$ là bất khả qui $mod \ p$ thì $P(x)$ là bất khả qui. Điều ngược lại của định lí nói chung không đúng.

Chứng minh

Giả sử $P(x)=a_nx^n+a_{n-1}x^{n-1}+…+a_1x+a_0 \in \mathbb{Z}[x]$ và $p$ không là ước của $a_n$.

Giả sử $P(x)=f(x).g(x)$ với $f, g \in \mathbb{Z}[x]$ với $deg \ f, g \ge 1$.

Khi đó $\overline{P}=\overline{f}. \overline{g}$. Do $p$ không là ước của $a_n$ nên bậc của các đa thức $P, f, g$ không thay đổi sau khi rút gọn theo modulo $p$.

Điều này chứng tỏ $\overline{P}$ khả qui theo modulo $p$, vô lí. Ta có điều phải chứng minh.

Ngược lại dễ thấy đa thức $P(x)=x^4+1$ bất khả qui trên $\mathbb{Z}[x]$ nhưng khả qui modulo $p$ với mọi số nguyên tố $p$.

Ví dụ 4.1: Chứng minh đa thức $P(x)=x^5+4x^4+2x^3+5x^2-7$ là bất khả qui.

Giải
  •  Rút gọn theo modulo 2 ta được $\overline{P}(x)=x^5+x^2+1.$
  •  Giả sử $\overline{P}(x)=f(x). g(x)$, với $f, g \in \mathbb{Z}_2$.
  •  Nếu $deg \ f=1$ hoặc $deg \ g=1$ dễ thấy là vô lí vì $\overline{P}$ không có nghiệm trong $\mathbb{Z}_2$.
  •  Suy ra $\overline{P}(x)=(x^2+ax+b)(x^3+cx^2+dx+e)$, với $a,b,c,d,e \in \mathbb{Z}_2$. Đồng nhất hệ số hai vế ta được điều vô lí. Từ đó suy ra điều phải chứng minh.

Ta có thể liệt kê ra các đa thức bất khả qui modulo 2 trong một số trường hợp bậc nhỏ như sau

  • Trường hợp $n=1$ gồm các đa thức: $x, x+1$.
  •  Trường hợp $n=2$ chỉ gồm một đa thức: $1+x+x^2$.
  •  Trường hợp $n=3$ gồm các đa thức: $1+x+x^3, 1+x^2+x^3$.
  •  Trường hợp $n=4$ gồm các đa thức: $1+x+x^4, 1+x+x^2+x^3+x^4$.
  •  Trường hợp $n=5$ gồm các đa thức:

$1+x+x^2+x^4+x^5$,

$1+x+x^3+x^4+x^5$,

$1+x^2+x^3+x^4+x^5$,

$1+x+x^2+x^3+x^5$,

$1+x^3+x^5, 1+x^2+x^5.$

5. Tiêu chuẩn Eisenstein và một số dạng mở rộng

Khi kiểm tra đa tính bất khả qui của một đa thức trên $\mathbb{Z}[x]$ tiêu chuẩn Eisenstein cung cấp cho ta một công cụ hiệu quả.

Định lí 5.1 (Tiêu chuẩn Eisenstein): Cho đa thức $P(x)= \sum \limits_{i=0}^na_ix^i \in \mathbb{Z}[x], a_n \ne 0$. Khi đó nếu tồn tại số nguyên tố $p$ thỏa đồng thời các điều kiện

  • $p$ không là ước của $a_n$;
  • $p$ là ước của $a_i$ với mọi $i\in \{1,2,…,n-1\}$;
  • $p^2$ không là ước của $a_0.$

Khi đó $P(x)$ là đa thức bất khả qui trên $\mathbb{Q}[x].$

Chứng minh

Có rất nhiều cách chứng minh cho tiêu chuẩn Eisenstein, ở đây ta sẽ trình bày chứng minh bằng cách rút gọn theo một modulo $p$ nguyên tố bất kì.

Giả sử $f$ khả qui, tức $f(x)=g(x).h(x)$, với $f, g \in \mathbb{Z}[x]$ và $deg \ f, g \ge 1.$ Rút gọn theo modulo $p$ nguyên tố ta được đẳng thức trong $\mathbb{Z}_p[x]$ dưới dạng $$\overline{f}=\overline{g}. \overline{h}.$$

Từ điều kiện $p$ là ước của $a_0, …, a_{n-1}$ nhưng không là ước của $a_n$ ta suy ra $\overline{f}=\overline{a_n}x^n.$

Từ đó suy ra $\overline{g}= \overline{b_k}x^k, \overline{h}=\overline{b_m}x^m$. Điều này có nghĩa là $\overline{b_0} \equiv…\equiv \overline{b_{k-1}} \equiv \overline{c_0} \equiv… \equiv \overline{c_{m-1}} \equiv 0 \ mod(p)$. Nhưng khi đó $a_0=b_0c_0 \equiv0 \ (mod \ p^2)$ (vô lí). Từ đó ta có điều phải chứng minh. $\square$

Ví dụ 5.1: Chứng minh đa thức $P(x)=x^4-x^2+2x+1$ bất khả qui trên $\mathbb{Z}$.

Giải

Đặt $Q(x)=P(x+1)=x^4+3x^3+3x^2+3x+3$. Khi đó theo tiêu chuẩn Eisenstein với $p=3$ ta có điều phải chứng minh.

Định lí 5.2 (Dạng mở rộng thứ nhất của tiêu chuẩn Eisenstein):

Cho $f(x)=a_nx^n+a_{n-1}x^{n-1}+…+a_1x+a_0 \in \mathbb{Z}[x]$. Giả sử tồn tại số nguyên tố $p$ thỏa mãn với một số tự nhiên $k \le n$ nào đó mà

  • p không là ước của $a_k$;
  • $p$ là ước của $a_0, …, a_{k-1}$;
  • $p^2$ không là ước của $a_0$.

Thế thì $f(x)$ có một nhân tử bất khả qui bậc $ \ge k$ ( và do đó nếu không bất khả qui sẽ có một nhân tử bậc $\le n-k$)

Chứng minh: Bạn đọc có thể tự chứng minh như trong trường hợp nguyên bản của định lí.

Ví dụ 5.2: Chứng minh đa thức $f(x)=x^{101}+101x^{100}+102$ là bất khả qui.

Giải

Áp dụng tiêu chuẩn Eisenstein mở rộng cho $p=2, k=100$ ta thấy, nếu $f$ là khả qui thì nó phải có một nhân tử bậc 1 và do đó $f$ phải có nghiệm hữu tỉ. Nói riêng vì hệ số bậc cao nhất bằng 1 nên nghiệm hữu tỉ này phải là nghiệm nguyên. Dễ thấy điều này là không xảy ra. bài toán được chứng minh xong.

 

Định lí 5.3 (Dạng mở rộng thứ hai của tiêu chuẩn Eisenstein):

Cho $f(x)=a_nx^n+a_{n-1}x^{n-1}+…+a_1x+a_0 \in \mathbb{Z}[x]$. Giả sử tồn tại số nguyên tố $p$ thỏa mãn với một số tự nhiên $k \le n$ nào đó mà

  • p không là ước của $a_n$;
  • $p$ là ước của $a_0, …, a_{n-1}$;
  • $p^2$ không là ước của $a_k$.

Thế thì, hoặc $f(x)$ là bất khả qui, hoặc $f$ có một nhân tử bất khả qui bậc $ \le k.$

Tương tự như trên, chứng minh được dành cho bạn đọc.

6. Các bài toán áp dụng

Bài tập 6.1 (IMO 1993): Cho số tự nhiên $n>1$. Chứng minh đa thức $f(x)=x^n+5x^{n-1}+3$ là bất khả qui trên $\mathbb{Z}[x]$.

Giải

Áp dụng dạng mở rộng thứ nhất của tiêu chuẩn Eisenstein với $p=3, k=n-1$ ta có điều phải chứng minh.

Bài tập 6.2 (China TST 1994): Cho số tự nhiên $n \ge 3$ và hai số nguyên tố $p, q$ phân biệt. Tìm tất cả các số nguyên $a$ sao cho đa thức $P(x)=x^n+ax^{n-1}+pq$ bất khả qui trên $\mathbb{Z}.$

Giải

Nếu $p|a$ hoặc $q|a$ thì theo tiêu chuẩn Eisenstein $P(x)$ là bất khả qui. Xét trường hợp $p,q$ đều không là ước của $a$. Giả sử $P$ khả qui, áp dụng dạng mở rộng thứ nhất của tiêu chuẩn Eisenstein ta suy ra $P(x)$ phải có nhân tử bậc 1 và do đó $P$ có một nghiệm nguyên $x_0$.

Từ đó suy ra $pq=-x_0^{n-1}(x_0+a)$. Vì $n \ge 3$ nên $pq \ \vdots \ x_0^2$ nhưng vì $p \ne q$ nên $x_0=\pm 1.$

Vì $1+a+pq=0$ và $(-1)^n+a(-1)^{n-1}+pq=0$ nên $a=-1-pq$ hoặc $a=1+(-1)^npq.$

Bài tập 6.3 (Rumani TST 1998): Chứng minh rằng đa thức $P(x)=(x^2+x)^{2^n}+1$ là bất khả qui với mọi số tự nhiên $n$.

Giải

Bằng qui nạp ta chỉ ra rút gọn modulo 2 thì đa thức đã cho trở thành $(x^2+x+1)^{2^n}$. Chú ý rằng đa thức $x^2+x+1$ là bất khả qui modulo 2.

Giả sử $P(x)$ khả qui, tức tồn tại hai đa thức $f,h$ đơn khởi với $f, g \in \mathbb{Z}[x], deg \ f,g \ge 1$ sao cho $P(x)=f(x).g(x)$. Khi đó $\overline{f}=(x^2+x+1)^k, \overline{g}=(x^2+x+1)^{2^n-k}$ và $$f(x)=(x^2+x+1)^k+2f_0(x), g(x)=(x^2+x+1)^{2^n-k}+2g_0(x),$$

với $f_0, g_0 \in \mathbb{Z}[x]$.

Gọi $j$ là một căn bậc 3 khác 1 của đơn vị. Thay $j$ vào đẳng thức $P(x)=f(x).h(x)$ ta được $$P(j)=g(j).h(j) \Leftrightarrow 2=4f_0(j)g_0(j).$$

Từ đó suy ra $f_0(j).g_0(j)=\dfrac{1}{2}$.

Do $f_0(j)g_0(j)$ luôn viết được dưới dạng $aj+b; a, b \in \mathbb{Z}$ và đẳng thức này không thể xảy ra. Ta có điều phải chứng minh.

Một số bài toán tương tự như sau:

Bài 1: Với $n \ge 1$ là số tự nhiên, chứng minh các đa thức sau là bất khả qui trên $\mathbb{Z}$

a) $P(x)=(x^3+x)^{2^n}-3$

b) $P(x)=(x^2+ax)^{2^n}+1$ với $ a \in \mathbb{Z}$

Bài 2: Cho $p$ là một số nguyên tố có dạng $4k+3$. Chứng minh rằng với mọi số nguyên dương $n$ đa thức $P(x)=(x^2+1)^n+p$ bất khả qui trên $\mathbb{Z}[x]$.

Bài 3: Cho $p$ là một số nguyên tố và $a$ là một số nguyên không chia hết cho $p$. Chứng minh đa thức $P(x)=x^p-x+a$ bất khả qui trên $\mathbb{Z}[x].$

Bài tập 6.4 (Japan 99): Chứng minh rằng đa thức $f(x)=(x^2+1^2)(x^2+2^2)…(x^2+n^2)+1$ là bất khả qui trên $\mathbb{Z}$

Giải

Giả sử $n \ge 2$ vì trường hợp $n=1$ là tầm thường và giả sử $f(x)=g(x).h(x)$ với $ f, g \in \mathbb{Z}[x]$ và $1 \le deg \ f, g \le n-1$. Khi đó $$1=f(\pm ki)=g(\pm ki) h(\pm ki).$$

Vì $f, g \in \mathbb{Z}[x]$ nên $g(\pm ki), h(\pm ki)$ có dạng $a + bi$ . Từ đó suy ra $$1=g(ki)h(ki)=1.1=(-1).(-1)=i.(-i)=(-i).i$$

Như vậy trong tất cả $4$ trường hợp ta đều có $g(ki)=\overline{h(ki)}=h(-ki),$ với $k=1,2,…,n$. Như vậy đa thức $P(x)=g(x)-h(-x)$ có $2n$ nghiệm phân biệt nhưng có bậc nhỏ hơn $2n$ nên là đa thức 0 và do đó $g(x)=h(-x)$. Suy ra $\deg(g)=\deg(h)=n $.

Vì $f$ đơn khởi nên $g,h$ cũng đơn khởi. Khi đó $g^2-h^2$ có bậc không quá $2n-1$ nhưng lại có ít nhất $2n$ nghiệm $ki$, với $i \in \{-n, -n-1,…,-1,1,…,n\}$ nên $g^2=h^2$.

Nhưng dễ thấy $g=-h$ không xảy ra do đó $g\equiv h$. Khi đó $$f(x)=g(x)^2 \Rightarrow f(0)=(g(0))^2=(n!)^2+1,$$ vô lí. Bài toán được chứng minh xong.

Ta có bài toán tổng quát hơn là: Cho $p$ là một số nguyên tố. Chứng minh rằng với mỗi số tự nhiên $n$ đa thức $$P(x)=(x^p+1^2)(x^p+2^2)…(x^p+n^2)+1$$ bất khả qui trên $\mathbb{Z}[x].$

Bài tập 6.5: Cho $m, n, a$ là các số nguyên dương và số nguyên tố $p$ thỏa mãn $p<a-1$. Chứng minh rằng đa thức $P(x)=x^m(x-a)^n+p$ bất khả qui trên $\mathbb{Z}$.

Giải

Giả sử $P(x)$ khả qui và $P(x)=G(x).H(x)$, với $G, H \in \mathbb{Z}[x]$. Vì $P(0)=p$ nên $|G(0)|=1$ hoặc $|H(0)|=1$. Không mất tổng quát ta giả sử $G(x)=x^k+a_{k-1}x^{k-1}+…+a_0$ và $|G(0)|=1, m+n-1 \ge k \ge 1.

Gọi $x_1,…,x_k$ là nghiệm của $G(x)$. Ta viết $G(x)$ dưới dạng $G(x)=(x-x_1)…(x-x_k)$, dẫn đến $|x_1…x_k|=1$ và $P(x_i)=0 \Leftrightarrow x_i^m(x_i-a)^n=-p$.

Cho $i=\overline{1,k}$ và nhân các vế của đẳng thức lại ta được $$ |G(x)|^n=p^k \ \text{nên} \ |G(a)|^n=p^k.$$

Mặt khác $P(a)=G(a).H(a)=p$ nên ta suy ra $|G(a)|=p$. Do đó $|G(a)-G(0)|=p \pm 1$ chia hết cho $a$.

Vì thế nên $p-1 \ge a$ hoặc $p+1 \ge a$ (mâu thuẫn với giả thiết $p<a-1$).

Vậy $P(x)$ là bất khả qui.

Bài tập 6.6 (Rumani 1999): Cho số nguyên $a$ và số nguyên dương $n$ và $p$ là một số nguyên tố thoả $p>|a|+1$. Chứng minh rằng đa thức $P(x)=x^n+ax+p$ bất khả qui trên $\mathbb{Z}[x]$.

Giải

Giả sử $P(x)=g(x).h(x)$, với $g, h \in \mathbb{Z}[x]$ và $1 \le deg f, deg g \le n-1$. Vì $P(0)=p=g(0).h(0)$ nên không mất tổng quát giả sử $g(0)=\pm 1, h(0)= \pm p.$

Khi đó $g(x)=\pm x^m+T(x)\pm 1, T \in \mathbb{Z}[x], deg T \le m-1.$

Gọi $z_1, z_2,\ldots,z_m$ là nghiệm của $g(x)$. Theo định lí Viet $1=|g(0)|=|z_1z_2…z_m|$ nên $|z_i| \le 1, i=1,2,…,m.$

Lại có $0=f(z_i)=z_i^n+az_i+p$ nên $$p=-z_i^n+az_i \le |z_i|^n+|a|.|z_i| \le 1+|a|,$$

vô lí. Vậy ta có điều phải chứng minh.

Bài tập 6.7: Cho $p, q$ là hai số nguyên tố lẻ sao cho $q$ không là ước của $p-1$ và gọi $a_1, a_2,…,a_n$ là các số nguyên phân biệt sao cho $q|(a_i-a_j)$ với mọi cặp $(i,j)$. Chứng minh rằng $$P(x)=(x-a_1)(x-a_2)…(x-a_n)-p$$ là bất khả qui trên $\mathbb{Z}[x]$ với mọi $n \ge 2.$

Giải

Giả sử $f$ khả qui trên $\mathbb{Z}[x]$. Khi đó tồn tại $Q(x), R(x) \in \mathbb{Z}[x]$ sao cho $Q(x)R(x)=P(x)$ và $1 \le deg Q(x), \le deg R(x).$ Nói riêng $degQ(x) \le \dfrac{n}{2}$.

Ta có $Q(a_i)=R(a_i)=-p,$ với $1 \le i \le n$ từ đó suy ra $Q(a_i), R(a_i) \in \{-1,1,-p,p \}$ với mọi $1 \le i \le n$. Với mọi hằng số $c$ đa thức $Q(x)-c$ nhận nhiều nhất $degQ(x) \le \dfrac{n}{2}$ nghiệm. Do đó $Q(a_i)$ nhận ít nhất hai giá trị phân biệt (và ít nhất $3$ giá trị phân biệt nếu $degQ(x)< \dfrac{n}{2}$).

Vì $q|(a_i-a_j)$ nên $q|(Q(a_i)-Q(a_j)$. Để ý rằng $q$ là số nguyên tố lẻ nên $Q(a_i)$ không thể nhận hai giá trị $1$ và $-1$ (vì nếu ngược lại thì $q|1-(-1)=2$). Tương tự $Q$ cũng không thể nhận hai giá trị là $p -p$ vì khi đó $R(a_i)$ nhận hai giá trị là $1,-1$.

Từ giả thiết $q$ không là ước của $p-1$ suy ra $Q(a_i)$ không thể nhận hai giá trị $1$ và $p$ hoặc $-1$ và $-p$. Do đó $Q(a_i)$ chỉ có thể nhận được nhiều nhất hai giá trị là $1$ và $-p$ hoặc $-1$ và $p$. Giả sử trường hợp đầu tiên xảy ra.

Vì $Q(a_i)$ chỉ nhận hai giá trị nên $degQ(a_i)=\dfrac{n}{2}$ và $Q$ nhận mỗi giá trị $1$ và $-p$ đúng $\dfrac{n}{2}$ lần. Chia tập $(a_i)_{i=1}^n$ thành hai tập $(b_i)_{i=1}^n$ và $(c_i)_{i=1}^n$ sao cho $Q(b_i)=1$ và $Q(c_i)=-p$. Khi đó $$Q(x)=(x-b_1)(x-b_2)…(x-b_{\frac{n}{2}})+1=(x-c_1)(x-c_2)…(x-c_{\frac{n}{2}})-p.$$

Vì ta cũng có $degR(x)=\dfrac{n}{2}$ nên $R(a_i)=-p$ khi $Q(a_i)=1$. Do đó $$R(x)=(x-b_1)(x-b_2)…(x-b_{\frac{n}{2}})-p=(x-c_1)(x-c_2)…(x-c_{\frac{n}{2}})+1.$$

Nhưng khi đó để ý rằng trong đẳng thức thứ nhất cho ta $Q(x)-R(x)=1+p$ còn đẳng thức thứ 2 cho ta $Q(x)-R(x)=-p-1$ điều đó dẫn đến $p=-1$. Vô lí. Bài toán được chứng minh xong.

Bài tập 6.8: Tìm tất cả các cặp số nguyên dương $(m,n)$ sao cho đa thức $$P(x,y)=(x+y)^2(mxy+n)+1$$ khả qui trên $\mathbb{Z}[x,y]$. Khi đó hãy phân tích $f$ thành các nhân tử bất khả qui.

Giải

Đặt $S=x+y$ ta viết lại $f$ dưới dạng $$f(x,S)=S^2(mx(S-x)+n)+1=-mS^2x^2+mS^3x+(nS^2+1).$$

Ta xem $f$ là một tam thức bậc 2 theo biến $x$. Khi đó $f$ khả qui khi và chỉ khi $f$ phân tích được thành tích của hai đa thức bậc nhất. Khi đó biệt thức $$\Delta =m^2S^6+4mS^2(nS^2+1)=mS^2(mS^4+4nS^2+4)$$ là một bình phương.

Dễ thấy điều này xảy ra khi và chỉ khi $m=n^2$, lúc này $$\Delta= (nS(nS^2+2))^2$$ và $f$ có hai nghiệm là $x=\dfrac{-1}{nS}$ hoặc $x=\dfrac{nS^2+1}{nS}$.

Khi đó $$f(x)=(nSx+1)(-nSx+nS^2+1)=(nx^2+nxy+1)(ny^2+nxy+1).$$

 

7. Bài tập tự luyện

Bài 1: Với $n \ge 2$ là một số nguyên và $r=\sqrt[n]{2}$. Chứng minh rằng không tồn tại các số hữu tỷ $a_0, a_1,…,a_{n-1}$ không đồng thời bằng $0$ sao cho $$ a_0+a_1r+a_2r^2+…+a_{n-1}r^{n-1}=0 $$

Bài 2: Tìm số nguyên dương $n$ nhỏ nhất sao cho đa thức $P(x)=x^{n-4}+4n$ có thể phân tích được thành tích của 4 đa thức hệ số nguyên và không là đa thức hằng.

Bài 3: Cho $P(x), Q(x)$ là hai đa thức đơn khởi, bất khả quy trên trường số hữu tỷ. Giả sử $P, Q$ có hai nghiệm tương ứng là $\alpha, \beta$ sao cho $\alpha +\beta$ là số hữu tỷ. Chứng minh $P^2(x)-Q^2(x)$ có nghiệm hữu tỷ.

Bài 4: Chứng minh đa thức $P(x)=(1+x+x^2+…+x^n)^2-x^n$ khả qui trên $\mathbb{Z}[x].$

Bài 5: Chứng minh rằng đa thức $P(x)=x^n+4$ khả qui trên $\mathbb{Z}$ khi và chỉ khi $n$ là bội của $4.$

Bài 6 (IMO Longlist 1989): Cho $n \ge 4$ và các số nguyên phân biệt $a_1,a_2,…,a_n$. Chứng minh đa thức $$P(x)=(x-a_1)(x-a_2)…(x-a_n)-2$$ bất khả qui trên $\mathbb{Q}[x].$

Bài 7 (VMO 2014): Cho $n$ là số nguyên dương. Chứng minh rằng đa thức $P(x)=(x^2-7x+6)^n+13$ không thể biểu diễn được thành tích của $n+1$ đa thức khác hằng với hệ số nguyên.

Bài 8: Chứng minh rằng đa thức $x^n-x-1$ bất khả qui trên $\mathbb{Q}[x]$, với mọi $n \ge 2.$

Bài 9: Cho $n>m>1$ là hai số nguyên lẻ. Chứng minh đa thức $P(x)=x^n+x^m+x+1$ bất khả qui trên $\mathbb{Z}[x]$.

Bài 10: Cho $p$ là số nguyên tố. Chứng minh rằng đa thức $$P(x)=x^{p-1}+2x^{p-2}+…+(p-1)x+p$$ bất khả qui trên $\mathbb{Z}$.

Bài 11: Cho đa thức $P(x)=a_nx^n+a_{n-1}x^{n-1}+…+a_1x+a_0 \in \mathbb{Z}[x], (a_n \ne 0, n \ge 2)$. Chứng minh rằng tồn tại vô số số nguyên tố $k$ sao cho đa thức $P(x)+k$ bất khả qui.

Bài 12: Tìm tất cả các số nguyên $n$ sao cho đa thức $P(x)=x^5-nx-n-2$ là khả qui trên $\mathbb{Z}[x]$.

Bài 13: Cho $p$ là một số nguyên tố và $n$ là một số nguyên nhỏ hơn 4. Chứng minh rằng nếu $a$ là một số nguyên không chia hết cho $p$ thì đa thức $P(x)=ax^n-px^2+px+p^2$ bất khả qui trên $\mathbb{Z}[x].$

Bài 14: Cho $p$ là số nguyên tố. Chứng minh rằng đa thức $P(x)=x^p+(p-1)!$ bất khả qui trên $\mathbb{Z}[x]$.

Bài 15: Tồn tại hay không đa thức $f \in \mathbb{Q}[x]$ sao cho $f(1) \ne -1$ và $x^nf(x)+1$ là khả qui với mọi $n \in \mathbb{N}$.

Bài 16: Cho $a$ là một số nguyên dương và $p \ge 2 $ là một số nguyên tố thỏa mãn $(a,p)=1$. Chứng minh rằng đa thức $P(x)=x^p-mx+a$ bất khả qui trên $\mathbb{Z}[x]$ với $m \equiv \ 1 \ (mod \ p)$.

Bài 17: Cho $p$ là một số nguyên tố lẻ. Chứng minh đa thức $P(x)= \sum \limits_{i=0}^{p-2}(p-1-i)x^i$ bất khả qui trên $\mathbb{Q}[x].$

Bài 18 (Rumani TST 2003): Cho $P(x) \in \mathbb{Z}[x]$ là một đa thức monic bất khả qui trên $\mathbb{Z}[x]$ sao cho $P(0)$ không là số chính phương. Chứng minh rằng $Q(x)=P(x^2)$ cũng bất khả qui trên $\mathbb{Z}[x].$

Bài 19 (China TST 2006): Cho số nguyên $n \ge 2$. Chứng minh rằng tồn tại đa thức $P(x)=x^n+a_{n-1}x^{n-1}+…+a_1x+a_0$ thỏa mãn

a) $a_0, a_1,…,a_{n-1}$ khác 0.

b) $P(x)$ bất khả qui.

c) Với mọi số nguyên $x$ thì $|P(x)|$ không là số nguyên tố.

Bài 20: Biết $f \in \mathbb{Z}[x]$ là một đa thức bất khả qui có bậc lẻ và lớn hơn 3. Giả sử rằng các nghiệm của $P$ đều có modun lớn hơn 1 và $f(0)$ không có ước chính phương. Chứng minh rằng đa thức $g(x)=f(x^3)$ cũng là đa thức bất khả qui.

Bài 21: Cho $f \in \mathbb{Z}[x]$ là một đa thức monic với bậc lớn hơn 1. Giả sử $f(x^n)$ bất khả qui trên $\mathbb{Z}[x]$ với mọi $n \ge 2$. Hỏi $f$ có bất khả qui trên $\mathbb{Z}[x]$ hay không?

Bài 22: Cho $1 \ne f \in \mathbb{Z}[x]$ sao cho có vô hạn số nguyên $a$ thỏa $f(x^2+ax)$ bất khả qui trên $\mathbb{Q}[x]$. Hỏi $f$ có bất khả qui trên $\mathbb{Q}[x]$ hay không?

Bài 23: Cho $f(x) \ne \pm x$ là một đa thức bất khả qui trên $\mathbb{Z}[x]$. Hỏi $f(x.y)$ có bất khả qui trên $\mathbb{Z}[x,y]$ hay không?

Tài liệu tham khảo

[1] Nguyễn Tiến Quang, NXB Giáo dục, Đại số đại cương

[2] Đoàn Duy Cường, 2015, Bài giảng bồi dưỡng giáo viên chuyên toán năm

[3] Nguyễn Chu Gia Vượng,2015,  Đa thức bất khả qui

[4] Exploration-Creativity 2012,  Irreducible polynomials

[5] Yufei Zhao, Integer polynomial 

[6] Dusan Djukic, Polynomials in one variable 

[7] Gabriel D.Carroll, Polynomials 

[8] Victor V.Prasolov, Polynomials

[9] Titu Andresscu, Gabriel Dospinescu, Problems from the book

[10] U298, Mathematical Reflections

[11] https://artofproblemsolving.com/community/c89